You are on page 1of 92

p?

Online PG Nepal
Questions Answers Explanations
[QAE file]

CEE MD/MS BASED MODEL TEST Designed to make you more


confident.
Magh 1, 2078 (Saturday) Master the high yield and most
frequently tested questions.
Simulate the real test in expected
pattern.

Review your Rank among others,


and re-design the extra efforts
required to ace the competitive
exams.
REAL CEE MD/MS ENTRANCE
Discover your weakness and focus
Magh 8, 2078 (Jan 22, 2022) on it.

Practice and Learn.

Designed by 100+ experts, and


near all toppers and rankers of
past 4 years of competitive exams.

Recommended by all toppers and


rankers of Last year entrance in

Contact details:
Email: onlinepgnepal@gmail.com
Facebook: fb.com/onlinepgnepal
This PDF file has been individualized for digital uniqueness based upon your Email address and Mobile Instagram: instagram.com/onlinepgnepal
number. Materials sent by the Online PG Nepal is for educational purpose only. Circulating the contents to anyone
other than the intended by Online PG Nepal in any form (print or digital) would be considered violation of copyright
Viber: 9801114177
protection and would attract legal actions. In view of the possibility of human error or changes in medical sciences, Mobile: 9801114177
we don't warrant that the information contained herein is in every respect accurate or complete.
ONLINE PG NEPAL QAE Magh 1

ONLINE PG NEPAL
CEE MD/MS BASED MODEL TEST
(Saturday, Magh 1, 2078)
www.onlinepgnepal.com
Visit our website for Online Model Tests and various PG preparation related stuffs and activities.

Time: 3 Hours Full marks: 200


No negative markings

How to get Daily Viber Update services: Step wise


• Save our number 9801114177 in your Mobile contact list as Online PG Nepal (Saving number is important).
• Send us "Hello, my name is ……" message in Viber number of 9801114177. Send your Email address, MBBS college.
• We will send you updates about various MD/MS entrance related activities, notices, questions/answers through Viber.
Any correction or explanation updates will be sent in Viber.

2
ONLINE PG NEPAL QAE Magh 1

Medicine
1. A 41-year-old female presents with band like bilateral tight headache. Headache last for around 2 hours when it
happens. She is on oral contraceptive pills for 13 years. There is no associated nausea, vomiting or photophobia seen.
Most likely type of headache she is suffering is:
a. Tension headache
b. Migraine
c. Cluster headache
d. Trigeminal neuralgia

Ans: a
Ref: Harrison 19th Edition, Page No: 2593
Tension-Type Headache
 The term tension-type headache (TTH) is commonly used to describe a chronic head-pain syndrome characterized
by bilateral tight, band-like discomfort.
 The pain typically builds slowly, fluctuates in severity, and may persist more or less continuously for many days.
The headache may be episodic or chronic (present >15 days per month).
 A useful clinical approach is to diagnose TTH in patients whose headaches are completely without accompanying
features such as nausea, vomiting, photophobia, phonophobia, osmophobia, throbbing, and aggravation with
movement. Such an approach neatly separates migraine, which has one or more of these features and is the main
differential diagnosis, from TTH.

Primary Headache Disorders


Headache Clinical Features Treatment
Type
Tension-  Bilateral headache described as a  Abortive
type o "band-like" o NSAIDs
headache o non-throbbing o acetaminophen
o no associated  Prophylactic
features o biofeedback and relaxation
 Duration therapy
o > 30 minutes o amitriptyline
 usually 4-6 hours
Migraine  Unilateral (or bilateral) headache that is  Abortive
headache o throbbing or pulsatile o NSAIDs
o worse with activity o acetaminophen
 Can be associated with o triptans
o photophobia o ergots
o phonophobia o antiemetics
o auras that can be  metoclopramide
 motor  chlorpromazine
 sensory  Prochlorperazine
 visual  Prophylactic
o nausea and vomiting o biofeedback and relaxation
 Triggers therapy
o odors o propranolol

3
ONLINE PG NEPAL QAE Magh 1

o lights o valproic acid


o foods o calcium channel blockers
o weather o topiramate
o menses o amitriptyline
 Duration
o 4-72 hours
Cluster  Unilateral and repetitive stab-like  Abortive
headache periorbital headaches that are associated with o 100% oxygen
o ipsilateral autonomic symptoms such o triptans
as  Prophylactic
 lacrimation o verapamil
 rhinorrhea o topiramate
 Horner's syndrome
o circadian periodicity
o pain being severely debilitating
 Duration
o 15 minutes - 3 hours

2. A 5-year-old male is brought to ER of Seti Provincial Hospital with brief lapses of consciousness and fall few hours
back. There was no injury in the body of the patient and without any complaints now. Which of the following does not
hold true in such case?
a. Such episodes may occur hundreds of times per day, but the child may be unaware of or unable to convey their
existence.
b. Often reported as “daydreaming” and a decline in school performance recognized by a teacher.
c. EEG shows generalized, symmetric, 3-Hz spike-and-wave discharge that begins and ends suddenly, superimposed
on a normal EEG background
d. Hypoventilation tends to provoke these electrographic discharges

Ans: d
Ref: Harrison 19th Edition, Page No: 2543
Typical Absence Seizures
 Typical absence seizures are characterized by sudden, brief lapses of consciousness without loss of postural
control.
 The seizure typically lasts for only seconds, consciousness returns as suddenly as it was lost, and there is no
postictal confusion. Although the brief loss of consciousness may be clinically inapparent or the sole manifestation
of the seizure discharge, absence seizures are usually accompanied by subtle, bilateral motor signs such as rapid
blinking of the eyelids, chewing movements, or small-amplitude, clonic movements of the hands.
 Typical absence seizures are associated with a group of genetically determined epilepsies with onset usually in
childhood (ages 4–8 years) or early adolescence and are the main seizure type in 15–20% of children with
epilepsy.
 The seizures can occur hundreds of times per day, but the child may be unaware of or unable to convey their
existence.
 Because the clinical signs of the seizures are subtle, especially to parents who may not have had previous
experience with seizures, it is not surprising that the first clue to absence epilepsy is often unexplained
“daydreaming” and a decline in school performance recognized by a teacher.

4
ONLINE PG NEPAL QAE Magh 1

 The electrophysiologic hallmark of typical absence seizures is a generalized, symmetric, 3-Hz spike-and-wave
discharge that begins and ends suddenly, superimposed on a normal EEG background. Periods of spike-and-wave
discharges lasting more than a few seconds usually correlate with clinical signs, but the EEG often shows many
more brief bursts of abnormal cortical activity than were suspected clinically.
 Hyperventilation tends to provoke these electrographic discharges and even the seizures themselves and is
routinely used when recording the EEG.

3. A 28-year-old male is brought to the ER of Sahid Memorial Hospital with confusion. The patient was hit by a car,
which ran away and details of the on-site history is not available. However, no loss of consciousness was observed. On
CT scan, which of the following area can have hemorrhage?
a. Anterior temporal lobes
b. Brain stem
c. Putamen
d. Thalamus

Ans: a
Ref: Harrison 19th Edition, Page No: 2583

4. A 60-year-old male is confused and repeatedly ask about his or her location in place and time. However, this happens
for few hours in a day and patient has normal memory. Patient cannot recall the episode of forgetfulness he had. Most
likely diagnosis of the patient is:
a. Alzheimer’s disease
b. Transient ischemic attack
c. Transient global amnesia
d. Vascular dementia

Ans: c
Ref: Harrison 19th Edition, Page No: 2608
Transient global amnesia (TGA)

5
ONLINE PG NEPAL QAE Magh 1

 Characterized by the sudden onset of a severe episodic memory deficit, usually occurring in persons over the age
of 50 years.
 Often the amnesia occurs in the setting of an emotional stimulus or physical exertion. During the attack, the
individual is alert and communicative, general cognition seems intact, and there are no other neurologic signs or
symptoms.
 The patient may seem confused and repeatedly ask about his or her location in place and time.
 The ability to form new memories returns after a period of hours, and the individual returns to normal with no
recall for the period of the attack.
 Frequently no cause is determined, but cerebrovascular disease, epilepsy (7% in one study), migraine, or cardiac
arrhythmias have all been implicated.
 Approximately one-quarter of patients experience recurrent attacks. Rare instances of permanent memory loss
have been reported in patients with TGA-like spells, usually representing ischemic infarction of the hippocampus
or dorsomedial thalamic nucleus bilaterally.
 Seizure activity due to AD should always be suspected in this syndrome.

5. A 42-year-old female visits you for with body ache and weakness of limbs. On examination, you take the patient's
middle finger and flick the distal phalanx, her thumb contracts in response. What sign has been elicited?
a. Chvostek’s
b. Glabellar
c. Hoffman’s
d. Tinel’s

Ans: c
Ref: Hoffmann Sign by Eric Whitney; Sunil Munakomi; StatPearls.
The Hoffman sign is an involuntary flexion movement of the thumb and or index finger when the examiner flicks the
fingernail of the middle finger down. The reflexive pathway causes the thumb to flex and adduct quickly.
A positive Hoffman sign can be indicative of an upper motor neuron lesion and corticospinal pathway dysfunction likely
due to cervical cord compression. However, up to 3% of the population has been found to have a positive Hoffman
without cord compression or upper motor neuron disease.

 Here, you have elicited a positive Hoffman’s reflex suggestive of upper motor neurone disease. It would have been
negative if the thumb had not contracted in response to flicking the patient’s distal phalanx.
 Chvostek’s sign is contraction of the face on stimulation of the facial nerve over the masseter. This is seen in
hypocalcaemia.
 The glabellar tap is an insensitive test for Parkinsonism where the doctor taps above the bridge of the nose and
the patient continues to blink. A normal response is to desensitize to the stimulus and stop blinking.
 Tinel’s sign can be elicited by tapping a nerve such as the ulnar nerve at the elbow, resulting in a tingling sensation
in the distribution of the nerve. This is a sign of nerve compression. It is also useful in carpal tunnel syndrome by
tapping over the median nerve at the wrist.

6. A blood thinner was started in a patient as prophylaxis and treatment of venous thrombosis and its extension. The
drug was initially developed as rodenticide. This drug acts by inhibiting which of the following:
a. Vitamin K epoxide reductase (VKOR)
b. Vitamin K reductase
c. Vitamin K carboxylase

6
ONLINE PG NEPAL QAE Magh 1

d. Vitamin K hydrolase

Ans: a
Ref: Harrison 19th Edition, Page No: 754
Warfarin
 A water-soluble vitamin K antagonist initially developed as a rodenticide, warfarin is the coumarin derivative most
often prescribed in North America.
 Like other vitamin K antagonists, warfarin interferes with the synthesis of the vitamin K–dependent clotting
proteins, which include prothrombin (factor II) and factors VII, IX, and X.
 The synthesis of the vitamin K–dependent anticoagulant proteins, proteins C and S, is also reduced by vitamin K
antagonists.
 It inhibits vitamin K epoxide reductase (VKOR) thereby interfering with the synthesis of the vitamin K dependent
clotting proteins (factor II, VII, IX, and X). Synthesis of vitamin K dependent anticoagulant proteins (proteins C & S)
is also reduced by vitamin K antagonists.

7. Which of the following is a fibrin-specific plasminogen activators?


a. Streptokinase
b. Alteplase
c. Anistreplase
d. Urokinase

Ans: b
Ref: Harrison 19th Edition, Page No: 758
 Plasminogen activators that preferentially activate fibrin-bound plasminogen are considered fibrin-specific.
 In contrast, nonspecific plasminogen activators do not discriminate between fibrin-bound and circulating
plasminogen.
 Activation of circulating plasminogen results in the generation of unopposed plasmin that can trigger the systemic
lytic state.
 Alteplase and its derivatives are fibrin-specific plasminogen activators, whereas streptokinase, anistreplase, and
urokinase are nonspecific agents. Alteplase, tenecteplase and reteplase are fibrin-specific plasminogen activators
while streptokinase, anistreplase and urokinase are nonspecific plasminogen activators.

8. Which of the following represents the initial lesion of atherosclerosis?


a. Necrotic core
b. Fatty streak
c. Microscopic breaches in endothelial integrity
d. Microthrombi rich in platelets

Ans: b
Ref: Harrison 19th Edition, Page No: 291e-1
 An integrated view of experimental results in animals and studies of human atherosclerosis suggests that the
“fatty streak” represents the initial lesion of atherosclerosis.
 These early lesions most often seem to arise from focal increases in the content of lipoproteins within regions of
the intima. In particular, the fraction of lipoproteins related to low-density lipoprotein (LDL) that bear
apolipoprotein B appears causally related to atherosclerosis.

7
ONLINE PG NEPAL QAE Magh 1

 This accumulation of lipoprotein particles may not result simply from increased permeability, or “leakiness,” of the
overlying endothelium.
 Rather, the lipoproteins may collect in the intima of arteries because they bind to constituents of the extracellular
matrix, increasing the residence time of the lipid-rich particles within the arterial wall.
 Lipoproteins that accumulate in the extracellular space of the intima of arteries often associate with proteoglycans
of the arterial extracellular matrix, an interaction that may slow the egress of these lipid-rich particles from the
intima.

9. Apolipoprotein that is essential for assembly & secretion of chylomicrons is:


a. Apo B100
b. Apo B48
c. Apo E
d. Apo AI

Ans: b
Ref: Harrison 19th Edition, Page No: 2346
ApoB is the major structural protein of chylomicrons, VLDLs, IDLs, & LDLs - apoB-48 in chylomicron and apoB-100 in VLDL,
IDL & LDL.

8
ONLINE PG NEPAL QAE Magh 1

10. Major determinant of total coronary resistance is found in:


a. Large epicardial arteries (R1)
b. Prearteriolar vessels (R2)
c. Arteriolar and intramyocardial capillary vessels (R3)
d. R2 + R3

Ans: d
Ref: Harrison 19th Edition, Page No: 1578
About 75% of the total coronary resistance to flow occurs across three sets of arteries:
 Large epicardial arteries (Resistance 1 = R1),
 Prearteriolar vessels (R2), and
 Arteriolar and intramyocardial capillary vessels (R3).
In the absence of significant flow-limiting atherosclerotic obstructions, R1 is trivial; the major determinant of coronary
resistance is found in R2 and R3.

11. Commotio cordis is due to:


a. Hypokalemia
b. Blunt trauma to chest
c. Hypomagnesemia
d. Myocardial infarction

Ans: b
Ref: Harrison 19th Edition, Page No: 289e-3
 Blunt, nonpenetrating, often innocent-appearing injuries to the chest may trigger ventricular fibrillation even in
absence of overt signs of injury. This syndrome, referred to as commotio cordis, occurs most often in adolescents
during sporting events (e.g., baseball, hockey, football, and lacrosse) and probably results from an impact to the
chest wall overlying the heart during the susceptible phase of repolarization just before the peak of the T wave.
Survival depends on prompt defibrillation.
 Sudden emotional or physical trauma, even in the absence of direct cardiac trauma, may precipitate a transient
catecholamine-mediated cardiomyopathy referred to as tako-tsubo syndrome or the apical ballooning syndrome.
 Commotio cordis is an often a lethal disruption of heart rhythm that occurs due to blunt trauma to precordial area
of chest at critical time of cardiac cycle. This usually occurs during sport injury leading to fatal ventricular
fibrillation.

12. A patient presents with dry cough for 4 months, hilar and paratracheal lymphadenopathy with normal lung
parenchyma in CXR. What is the probable diagnosis?
a. Sarcoidosis
b. Lymphoma
c. Tuberculosis
d. Interstitial pneumonia

Ans: a
Ref: Miller BH, Rosado-de-christenson ML, Mcadams HP et-al. Thoracic sarcoidosis: radiologic-pathologic correlation.
Radiographics. 1995;15 (2): 421-37 / Chest Radiology. Lippincott Williams & Wilkins. (2012) ISBN:146980204X.

9
ONLINE PG NEPAL QAE Magh 1

Garland triad, also known as the 1-2-3 sign or pawnbroker's sign, is a lymph node enlargement pattern on chest
radiographs which has been described in sarcoidosis:
 Right paratracheal nodes
 Right hilar nodes
 Left hilar nodes
Hilar lymphadenopathy is symmetrical and usually massive. These so-called potato nodes typically do not abut the cardiac
border which distinguishes the nodal enlargement from lymphoma.
Sarcoid granulomata mainly occur in a distribution along the lymphatics in the bronchovascular sheath and, to a lesser
extent, in the interlobular septa and subpleural lung regions. This distribution is one of the most helpful features in
recognizing sarcoidosis pathologically and is responsible for the high rate of success in diagnosis by bronchial and
transbronchial biopsies.
Clinical presentation is variable. Approximately 50% of patients are asymptomatic. The remainder present with either
respiratory symptoms (e.g. cough and dyspnea) or skin changes (e.g. erythema nodosum, lupus pernio, scars, plaques).

13. A 71-year-old male has difficulty while looking downstairs. This occurs due to the lesion of:
a. Oculomotor nerve
b. Trigeminal nerve
c. Trochlear nerve
d. Abducens nerve

Ans: c
Ref: Harrison 19th Edition, Page No: 209
 Trochlear Nerve The fourth cranial nerve originates in the midbrain, just caudal to the oculomotor nerve complex.
Fibers exit the brainstem dorsally and cross to innervate the contralateral superior oblique.
 The principal actions of this muscle are to depress and intort the globe. A palsy therefore results in hypertropia
and excyclotorsion. The cyclotorsion seldom is noticed by patients. Instead, they complain of vertical diplopia,
especially upon reading or looking down.
 The vertical diplopia also is exacerbated by tilting the head toward the side with the muscle palsy and alleviated by
tilting it away. This “head tilt test” is a cardinal diagnostic feature.
 Isolated trochlear nerve palsy results from all the causes listed above for the oculomotor nerve except aneurysm.
The trochlear nerve is particularly apt to suffer injury after closed head trauma. The free edge of the tentorium is
thought to impinge on the nerve during a concussive blow.
 Most isolated trochlear nerve palsies are idiopathic and hence are diagnosed by exclusion as “microvascular.”
 Spontaneous improvement occurs over a period of months in most patients.
 A base-down prism (conveniently applied to the patient’s glasses as a stick-on Fresnel lens) may serve as a
temporary measure to alleviate diplopia. If the palsy does not resolve, the eyes can be realigned by weakening the
inferior oblique muscle.

14. Which of the following ECG finding is first seen in acute pulmonary embolism?
a. S1Q3T3
b. ST-elevation
c. Sinus tachycardia
d. T wave inversion

Ans: c

10
ONLINE PG NEPAL QAE Magh 1

Ref: Harrison 19th Edition, Page No: 1633


 Elevated cardiac biomarkers: Serum troponin and plasma heart-type fatty acid–binding protein levels increase
because of RV microinfarction. Myocardial stretch causes release of brain natriuretic peptide or NT-pro-brain
natriuretic peptide.
 Electrocardiogram: The most frequently cited abnormality, in addition to sinus tachycardia, is the S1Q3T3 sign: an
S wave in lead I, a Q wave in lead III, and an inverted T wave in lead III.
 This finding is relatively specific but insensitive. RV strain and ischemia cause the most common abnormality, T-
wave inversion in leads V1 to V4.

15. 4-9 ribs notching is characteristic of


a. Co-arctation of aorta
b. Tetralogy of Fallot
c. Ventricular septal defect
d. Patent ductus arteriosus

Ans: a
Ref: Harrison 19th Edition, Page No: 1525
Coarctation of the aorta
 Narrowing or constriction of the lumen of the aorta may occur anywhere along its length but is most common
distal to the origin of the left subclavian artery near the insertion of the ligamentum arteriosum.
 Coarctation occurs in ~7% of patients with CHD, is more common in males than females, and is particularly
frequent in patients with gonadal dysgenesis (e.g., Turner’s syndrome).
 Clinical manifestations depend on the site and extent of obstruction and the presence of associated cardiac
anomalies, most commonly a bicuspid aortic valve.
 Circle of Willis aneurysms may occur in up to 10%.
 Most children and young adults with isolated, discrete coarctation are asymptomatic. Headache, epistaxis, chest
pressure, and claudication with exercise may occur, and attention is usually directed to the cardiovascular system
when a heart murmur or hypertension in the upper extremities and absence, marked diminution, or delayed
pulsations in the femoral arteries are detected on physical examination.
 Enlarged and pulsatile collateral vessels may be palpated in the intercostal spaces anteriorly, in the axillae, or
posteriorly in the interscapular area.
 The upper extremities and thorax may be more developed than the lower extremities.
 A midsystolic murmur over the left interscapular space may become continuous if the lumen is narrowed
sufficiently to result in a high-velocity jet across the lesion throughout the cardiac cycle. Additional systolic and
continuous murmurs over the lateral thoracic wall may reflect increased flow through dilated and tortuous
collateral vessels.
 The ECG usually reveals LV hypertrophy.
 Chest x-ray may show a dilated left subclavian artery high on the left mediastinal border and a dilated ascending
aorta. Indentation of the aorta at the site of coarctation and pre- and poststenotic dilatation (the “3” sign)
along the left paramediastinal shadow are essentially pathognomonic.
 Notching of the third to ninth ribs, an important radiographic sign, is due to inferior rib erosion by dilated
collateral vessels.
 Two-dimensional echocardiography from suprasternal windows identifies the site of coarctation;
 Doppler quantitates the pressure gradient. Transesophageal echocardiography and MRI or CT allow visualization
of the length and severity of the obstruction and associated collateral arteries.

11
ONLINE PG NEPAL QAE Magh 1

 In adults, cardiac catheterization is indicated primarily to evaluate the coronary arteries or to perform catheter-
based intervention (angioplasty and stent of the coarctation).
 The chief hazards of proximal aortic severe hypertension include cerebral aneurysms and hemorrhage, aortic
dissection and rupture, premature coronary arteriosclerosis, aortic valve failure, and LV failure; infective
endarteritis may occur on the coarctation site or endocarditis may settle on an associated bicuspid aortic valve,
which is estimated to be present in 50% of patients.

16. Piecemeal necrosis is present in


a. Chronic active hepatitis
b. Alcoholic hepatitis
c. Wilson disease
d. Indian childhood cirrhosis

Ans: a
Ref: Harrison 19th Edition, Page No: 2031
 Piecemeal necrosis of the liver is associated with a lymphocytic infiltrate into the adjacent parenchyma, and with
destruction of individual hepatocytes along the edges of the portal tract.
 It is a feature of viral hepatitis (especially chronic active hepatitis) as well as autoimmune hepatitis and
steatohepatitis.

17. Pinpoint pupil, pyrexia and paraplegia occur in


a. Multiple sclerosis
b. Cerebral infarction
c. Pontine hemorrhage
d. Hydrocephalus

Ans: c
Ref: Harrison 19th Edition, Page No: 2583
Pontine hemorrhages:
 Deep coma with quadriplegia often occurs over a few minutes. Typically, there is prominent decerebrate rigidity
and “pinpoint” (1 mm) pupils that react to light.
 There is impairment of reflex horizontal eye movements evoked by head turning (doll’s-head or oculocephalic
maneuver) or by irrigation of the ears with ice water.
Hyperpnea, severe hypertension, and hyperhidrosis are common. Most patients with deep coma from pontine
hemorrhage ultimately die, but small hemorrhages are compatible with survival.
Pontine hemorrhage, a form of intracranial hemorrhage, is most commonly due to long standing poorly-controlled chronic
hypertension.
It carries a very poor prognosis.
Clinical presentation
Patients present with sudden and precipitous neurological deficits. Depending on the speed at which the hematoma
enlarges and the exact location, presentation may include:
 Decreased level of consciousness (most common)
 Long tract signs including tetraparesis
 Cranial nerve palsies
 Seizures

12
ONLINE PG NEPAL QAE Magh 1

 Cheyne-stokes respiration

18. Digoxin is contraindicated in


a. Atrial fibrillation
b. Ventricular tachycardia
c. Heart failure
d. HOCM

Ans: d
Ref: Harrison 19th Edition, Page No: 473e-9 / Ref: Management of Hypertrophic Cardiomyopathy by Alan D.Enriquez MD;
Annals of Global Health, Volume 80, Issue 1, January–February 2014, Pages 35-45
Medications to be avoided or used with caution in Hypertrophic Cardiomyopathy
 Vasodilators such as nifedipine, nitrates, and angiotensin-converting enzyme inhibitors can cause venodilation, a
fall in systemic vascular resistance, or a combination of both. This can exacerbate LVOT obstruction and lead to
hypotension and worsening of heart failure symptoms.
 Similarly, digoxin should be avoided because of the positive inotropic effect.
 Diuretics, on the other hand, may be useful in patients with persistent symptoms despite treatment with β-
blockers or verapamil. However, they should be administered with caution in patients with LVOT obstruction.
Cardiac glycosides toxicity
 Drugs: Digoxin, endogenous cardioactive steroids, foxglove and other plants, toad skin secretions (Bufonidae spp.)
 Mechanism: Inhibition of cardiac Na+K+-ATPase membrane pump
 Clinical features: Physiologic depression; gastrointestinal, psychiatric, and visual symptoms; atrioventricular block
with or without concomitant supraventricular tachyarrhythmia; ventricular tachyarrhythmias; hyperkalemia in
acute poisoning. Toxicity occurs at lower drug levels in chronic poisoning than in acute poisoning.
 Treatment: Digoxin-specific antibody fragments for hemodynamically compromising dysrhythmias, Mobitz II or
third-degree atrioventricular block, hyperkalemia (>5.5 mEq/L; in acute poisoning only). Temporizing measures
include atropine, dopamine, epinephrine, and external cardiac pacing for bradydysrhythmia and magnesium,
lidocaine, or phenytoin, for ventricular tachydysrhythmia. Internal cardiac pacing and cardioversion can increase
ventricular irritability and should be reserved for refractory cases.

19. A 40-years old male was brought to the hospital with acute pain in the upper abdomen. Patient was in shock with
feeble pulse and tachycardia. There was tenderness present in the epigastrium. There is no blood in the gastric aspirate
and the patient felt better after aspiration. X-ray abdomen showed no free gas under the diaphragm. Investigations
revealed TLC 13500 serum bilirubin 3.1 mg and serum lipase 120 I.U. Ultrasound is normal. The most likely diagnosis is:
a. Acute cholecystitis
b. Acute pancreatitis
c. Acute appendicitis
d. Acute hepatitis

Ans: d
 Normal lipase and normal USG
 High bilirubin level
 High TLC
Lipase is produced by the pancreas, liver, intestine, tongue, stomach, and many other cells. Lipase testing is indicated in
acute pancreatitis, as well as in the diagnosis of peritonitis, strangulated or infarcted bowel, and pancreatic cyst.

13
ONLINE PG NEPAL QAE Magh 1

The reference range for lipase is 0-160 U/L or 0-160 U/L (SI units), although values depend on method.
The reference range for amylase is as follows:
 Serum test: Normal is 40-140 U/L
 Urine Test: Normal is 24-400 U/L

20. Which one of the following life - threatening congenital anomalies in the newborn presents with polyhydramnios,
aspiration pneumonia, excessive salivation and difficulty in passing a nasogastric tube?
a. Choanal atresia
b. Tracheo-esophageal fistula
c. Diaphragmatic hernia
d. Gastroschisis

Ans: b
Ref: SRB Surgery 4th Edition, Page No: 857
Tracheo-oesophageal fistula
Types
In 85% cases, it is a blind upper end with lower end communicating with trachea.
It may be associated with VACTER anomalies.
 V — Vertebral defects
 A — Anal atresia
 C — Cardiac defect (PDA/VSD)
 TE — Tracheo-esophageal fistula
 R — Radial hypoplasia and renal agenesis
Clinical Features
 TOF should be recognized within 24 hours of birth.
 Newborn baby regurgitates all feeds and there is continuous pouring of saliva from the mouth which is a
diagnostic feature.
 Cough and cyanosis.
 It is commonly associated with maternal hydramnios (50%).

21. A 50 years old male presents with obstructive symptoms. Biopsy of stomach reveals gastrointestinal stromal tumor
(GIST). Most appropriate market for GIST is:
a. CD-34
b. CD-117
c. CD-30
d. CD-10

Ans: b
Ref: Sabiston Surgery 20th Edition, Page No: 1270
 Stromal tumors arise from the interstitial cell of Cajal, an intestinal pacemaker cell of mesodermal descent.
 Three histologic types of stromal tumors are noted on the basis of their cellular appearance; tumors may be
fusiform (77%), epithelioid (8%), or mixed (15%).
 Stromal tumors are three to four times more frequent than malignant GISTs and are most commonly found in the
stomach (60%) and the jejunum and ileum (30%). They are rarely found in the duodenum (5%). More than 95% of

14
ONLINE PG NEPAL QAE Magh 1

stromal tumors express CD117, the c-kit proto-oncogene protein that is a transmembrane receptor for the stem
cell growth factor, and 70% to 90% express CD34, the human progenitor cell antigen.

22. A 62-year-old female has long history of diarrhea and colic pain on and off with mass in right iliac fossa. Most
probable diagnosis is:
a. Carcinoma rectum
b. Carcinoma cecum
c. Carcinoma sigmoid
d. Carcinoma transverse colon

Ans: b
Ref: SRB Surgery 4th Edition, Page No: 964
Adenocarcinoma
 Sigmoid colon (21%) is the most common site of malignancy after rectum (38%).
 In caecum it is 12% common.
A mass in the right iliac fossa is a common diagnostic problem encountered in clinical practice, requiring skill in diagnosis.
A swelling in the right iliac fossa may arise from the structures normally present in that region or from structures, which
are abnormally situated in the region. The mass could be intra-abdominal or extra-abdominal. The common swellings
which occur in the right iliac fossa are appendicular lump, carcinoma of the cecum, ileocecal tuberculosis and Crohn’s
disease (common in the West). Rare swellings are actinomycosis, ameboma, psoas abscess and lymph node masses.

23. In CECT, transverse colon is dilated to at least 8 cm with loss of haustral markings, with few pseudopolyps often
extending into the lumen. This is seen in:
a. Carcinoma colon
b. Gastrocolic fistula
c. Ulcerative colitis
d. Amoebic colitis

Ans: c
Ref: Sabiston Surgery 20th Edition, Page No: 1344
 Toxic megacolon is a serious life-threatening condition that can occur in patients with ulcerative colitis, Crohn’s
colitis, and infectious colitides such as pseudomembranous colitis, in which the bacterial infiltration of the walls of
the colon creates a dilation of the colon that progresses to the point of imminent perforation.
 This decompensation results in a necrotic thin-walled bowel in which pneumatosis can often be seen
radiographically.
 Although some patients with toxic megacolon have been successfully treated medically, a high rate of recurrence
with subsequent urgent operation has been reported.
 Aggressive preoperative stabilization is required, using volume resuscitation with crystalloid solutions to prevent
dehydration secondary to third-space fluid losses, stressdose steroids for patients previously receiving steroid
therapy, and broad-spectrum antibiotics.

24. Most common cause of death in Crohn’s disease is due to:


a. Sepsis
b. Thromboembolic complication
c. Electrolyte disturbance

15
ONLINE PG NEPAL QAE Magh 1

d. Malignancy

Ans: d
Ref: Mortality in Crohn's disease--a clinical analysis by Andrews HEADACHE; Q J Med. 1989 May;71(265):399-405.
Ref: Sabiston Surgery 20th Edition, Page No: 1266
 Sepsis and electrolyte imbalance have declined in importance and cancer of the digestive tract is now the most
common cause of related death.
 Standardized mortality rates in patients with Crohn’s disease are increased in those patients whose disease began
before the age of 20 years and in those who have had disease present for longer than 13 years. Long-term survival
studies have suggested that patients with Crohn’s disease have a death rate approximately two to three times
higher than that in the general population, which is most commonly related to chronic wound complications and
sepsis.
 Gastrointestinal cancer remains the leading cause of disease related death in patients with Crohn’s disease; other
causes of disease-related deaths include sepsis, thromboembolic complications, and electrolyte disorders.

25. Most common post-splenectomy infection is


a. Streptococcus pyogenes
b. Staphylococcus aureus
c. Streptococcus pneumoniae
d. Pseudomonas aeruginosa

Ans: c
Ref: Sabiston Surgery 20th Edition, Page No: 1556, 1557
 The immune functions of the spleen become obvious after splenectomy, when patients are noted to be
significantly at risk for infection. The most serious sequela is overwhelming post-splenectomy infection (OPSI),
with meningitis, pneumonia, or bacteremia.
 Older studies have demonstrated that the risk of OPSI is greatest within the first 2 years after splenectomy, but
recent studies have confirmed that a lifelong risk remains.
 One third of cases occur more than 5 years after surgery, with the overall incidence reported to be 3.2% to 3.5%.
For those who acquire OPSI, mortality is between 40% and 50%.
 The risk is greatest in patients with thalassemia major and sickle cell disease. OPSI is typically caused by
polysaccharide-encapsulated organisms, such as Streptococcus pneumoniae, Neisseria meningitidis, and
Haemophilus influenzae.
 These and other organisms are identified and bound by antibodies and complement components in preparation
for phagocytosis by macrophages in the spleen.
 After splenectomy, the antibodies continue to bind, but digestion by splenic macrophages is no longer possible.

26. Staghorn calculus is made of:


a. Oxalate
b. Phosphate
c. Uric acid
d. Cystine

Ans: b
Ref: SRB Surgery 4th Edition, Page No: 1090, 1091

16
ONLINE PG NEPAL QAE Magh 1

Renal stones:
 Oxalate stones (75%): Also called as mulberry stone as it is brown in colour, with sharp projections. It is invariably
calcium oxalate stone, shows envelope crystals in urine.
 Phosphate stones (10-15%): It is either calcium phosphate or calcium, magnesium, ammonium phosphate stone
usually occurring in an infected urine. It is smooth and white in colour. In alkaline urine it enlarges rapidly, filling
renal calyces taking their shape called as staghorn calculus. It is radio-opaque and attains a large size.
 Uric acid stones (5%) are smooth, hard, yellowish, multiple and radiolucent. They are seen in gout, hyper
uricosuria, altered purine metabolism.
 Urate stones
 Cystine stones (2%) occur in cystinuria where there is defective absorption of cystine from the renal tubules
(autosomal recessive condition). It is seen in young girls, occurs only in acidic urine. It is multiple, soft, yellow in
colour and the colour changes to greenish hue on exposure. It attains large size. It is radiopaque because it
contains sulphur.
 Xanthine stones are very rare, smooth, brick red in colour, due to altered xanthine metabolism. Here there is
deficiency in xanthine oxidase enzyme.
 Indigo stones: Very rare. Blue in colour.
 Struvite stone: It is compound of magnesium, ammonium phosphate mixed with carbonate. It occurs in presence
of ammonia and urea splitting organisms in urine, e.g. Proteus; Klebsiella.

Type of crystal and Shape of the crystal:


 Calcium oxalate monohydrate Dumbbell shaped
 Calcium oxalate dihydrate Envelope shaped
 Uric acid Yellowish of varying size and shape
 Cystine Hexagonal, very soft stones
 Triple stone Coffin lid shaped

27. Stipple sign in transitional cell carcinoma of the renal collecting system is best demonstrated by:
a. Intravenous urography
b. Retrograde pyeloureterography
c. Radionuclide scan
d. Ultrasound scan

Ans: b
Ref: Stipple sign (transitional cell carcinoma) by Frank Gaillard et al in Radiopedia.
The stipple sign refers to the pointillistic end-on appearance on intravenous pyelography or retrograde pyelography of
contrast material tracking into the interstices of a papillary lesion. Because the majority of transitional cell carcinomas
(TCC) have a papillary configuration, the presence of this sign should raise the suspicion of TCC. While the stipple sign is
best seen in large papillary bladder tumors, it can occur anywhere a urothelial tumor expresses papillary architecture.

28. Arteriovenous fistula can safely be ligated if the following is positive:


a. Allen’s test
b. Henle-Coenen sign
c. Trendelenburg test
d. Schwartz test

17
ONLINE PG NEPAL QAE Magh 1

Ans: a
Ref: SRB Surgery 4th Edition, Page No: 179
Allen’s test:
 It is done to find out the patency of radial and ulnar arteries. Both the arteries are compressed near the wrist and
allowed to blanch completely in one minute (In the mean time patient closes and opens the fi st several times for
further venous outflow).
 Palm appears pale and white.
 One of the arteries is released and colour of hand is noted. Normally hand will become pink and flushed in no
time; whereas in obstruction, the area will still remain pale.
 Other artery is also released and looked for changes in hand. Often test has to be repeated to get proper
information.
 Tests the adequacy of the blood supply to the hand from the radial and ulnar arteries and the arcade between
them. Allen’s test is used to know the integrity of palmar arch (patency of radial and ulnar arteries). If these are
patent, AVF can be safely ligated.

29. Lethal midline granuloma is:


a. Wagner’s granulomatosis
b. Extra nodal NK cell/T cell lymphoma nasal type
c. Syphilis of nasal septum
d. Tuberculosis of nasal septum

Ans: b
Ref: Lethal midline granuloma by Varuna Mallya; Indian Dermatol Online J. 2013 Jan-Mar; 4(1): 37–39.
 Lethal midline granuloma is a mid-facial necrotizing lesion that is characterized by destructive, mucosal lesions of
the upper aero digestive tract.
 The patients complain of rhinorrhea, epistaxis, nasal stuffiness, obstruction and pain. The underlying mucosa is
thickened and the patient usually develops extensive mid-facial destructive lesions, perforated nasal septum and
erosion of the nasal bone.
 The disease is localized to the upper aero-digestive tract at presentation but dissemination to distant sites may
occur. Constitutional symptoms may develop.
 Also referred to as polymorphic reticulosis, midline malignant reticulosis, Stewart s granuloma most of the
lethal midline granulomas are NK/T cell lymphomas.

30. Bastio surgery for refractory LV hypertrophy is:


a. Patch repair
b. MR repair
c. Ventriculectomy
d. Ventriculoplasty

Ans: c
Bentall’s operation:
 For aortic root aneurysm repair
 Reimplantation of coronary ostia into composite graft
Bastio surgery:
 For Left ventricular hypertrophy

18
ONLINE PG NEPAL QAE Magh 1

 Ventriculectomy is done

31. You are assisting with a total thyroidectomy for a patient who has a goitre and is experiencing compressive
symptoms. Surgeon asks you to name the artery supplying the thyroid gland, which originates from the external carotid
artery. Answer will be:
a. Inferior thyroid artery
b. Thyroid ima artery
c. Superior thyroid artery
d. Ascending pharyngeal artery

Ans: c
Ref: Sabiston Surgery 20th Edition, Page No: 884
The arterial supply to the thyroid gland consists of four main arteries, two superior and two inferior.
The superior thyroid artery is the first branch of the external carotid artery after the bifurcation of the common carotid
artery.
 The superior thyroid artery is the first branch of the external carotid artery and enters the upper pole of each
lateral lobe of the thyroid gland.
 The main arterial supply of the lower pole of the thyroid gland is the inferior thyroid artery originating from the
thyrocervical trunk, which is a branch of the first part of the subclavian artery.
 The thyroid ima artery, which is not always present, originates from either the aortic arch or the brachiocephalic
trunk and enters the lower aspect of the thyroid isthmus.
 The ascending pharyngeal artery is branch of the external carotid that do not supply the thyroid gland.

32. You are examining a patient and looking for the second intercostal place by finding the manubriosternal junction
(also known as the ‘angle of Louis’). From the list below, choose the vertebral level that corresponds to the
manubriosternal junction.
a. T1
b. T2
c. T3
d. T4

Ans: d
Ref: SRB Surgery 4th Edition, Page No: 1575
The sternum is flat, 15 to 20 cm long, and approximately 1.0 to 1.5 cm thick and comprises the manubrium, body, and
xiphoid.
The manubrium articulates with each clavicle and the first rib. The manubrium joins the body of the sternum at the angle
of Louis, which corresponds to the anterior aspect of the junction of the second rib. The angle of Louis is a superficial
anatomic landmark for the level of the carina. The anterior cartilaginous attachments of the true ribs to the sternum,
along with intercostal muscles and the hemidiaphragms, allow for movement of the ribs with respiration.
The manubriosternal junction, also known as the sternal angle or ‘angle of Louis’, is at the level of the second costal
cartilage towards the lower border of the T4 vertebra.

33. The nursing staff asks you to review an erect chest radiograph of a 60-year-old woman who has undergone open
colonic surgery for a pelvic mass 3 days ago. She is comfortable at rest. Her abdomen is distended, with absent bowel
sounds. Free air under the hemi-diaphragms is likely to be due to

19
ONLINE PG NEPAL QAE Magh 1

a. Perforated peptic ulcer


b. Anastomotic leakage
c. Perforated sigmoid diverticulum
d. A normal finding 4 days post laparotomy

Ans: d
The presence of free air under the diaphragm is not uncommon following open and laparoscopic surgery and is the most
likely explanation for this finding. This represents a normal finding 3 days post laparotomy.
Other less likely causes of free intraperitoneal air in this setting include anastomotic leakage, perforated sigmoid
diverticulum and perforated peptic ulcer; however, these are extremely unlikely in a patient who is otherwise well.

34. You see an 18-year-old lady in the theatre admission lounge who is due to have a removal of a right breast
fibroadenoma under general anaesthesia. You ask her whether she has had anything to eat or drink after midnight on
the same day as the operation. From the list below, which of the following correctly applies to preoperative starvation
in adults?
a. Patients should not eat solid food for 6 hours prior to a general anaesthetic
b. Patients may eat solid food up to 4 hours before a general anaesthetic
c. Patients should not eat solid food for 12 hours prior to a general anaesthetic
d. Patients may eat solid food up to 2 hours before a general anaesthetic

Ans: a
 The American Society of Anaesthesiologists (ASA) and the Association of Anaesthetists of Great Britain and Ireland
(AAGBI) have recommended that for adults solids and liquids should not be consumed by patients undergoing
elective surgical procedures involving general anaesthesia or sedation for 6 and 2 hours respectively, prior to their
surgical procedure.
 In addition, patients having regional or local anaesthetic procedures should follow the same ‘nil by mouth’ policy
as those scheduled for a general anaesthetic.
 For patients undergoing emergency surgical procedures that involve a general anaesthetic, nasogastric aspiration
is usually performed to decrease gastric contents and hence reduce the risk of pulmonary aspiration.

35. An initial primary survey of the chest is intended to quickly identify the following causes of cardiorespiratory
compromise, except
a. Flail chest
b. Cardiac tamponade
c. Tension pneumothorax
d. Pulmonary contusion

Ans: d
The ATLS guidelines teach a method of rapid assessment of the traumatized patient divided into a primary survey, which is
aimed at the identification of immediate threats to life, and a secondary survey, which aims to identify more minor
injuries.
Thoracic injuries which are an immediate threat to life include flail chest, tension pneumothorax, open pneumothorax,
massive haemothorax and cardiac tamponade.

20
ONLINE PG NEPAL QAE Magh 1

Pulmonary contusion is not an immediate threat to life, and therefore is not of initial concern during the primary survey.
However, it can complicate recovery as the injured lung can impair gas exchange and may be more susceptible to fluid
overload.

36. A patient is admitted in ER of Beer Hospital following a house fire in New Road. He has extensive partial and full
thickness burns over his arms, upper torso and neck. You note black carbon deposits around his nostrils and
oropharynx. Which of the following is the immediate priority?
a. Adequate analgesia
b. Sterile water irrigation
c. Intubation
d. Fluid resuscitation

Ans: c
 The subglottic airway is protected from thermal injury by the larynx.
 However, the supraglottic airway is susceptible to such injury and upper airway occlusion is a common
consequence of inhalational injury or extensive burns to the neck.
 Signs of inhalation injury include singed nasal hair, facial burns, carbon deposits around the nose and oropharynx,
hoarseness of voice, carbonaceous sputum and history of confinement in a burning environment.
 This patient is at high risk of complicating his airway, with evidence of inhalational injury combined with external
injury to the neck. Securing his airway is of highest priority as delay may lead to laryngeal oedema and need for
surgical airway placement.
 This patient will also require fluid resuscitation and transfer to a specialist burns centre. Water irrigation and
removal of all clothing is required to ensure removal of all burning material from the site of injury; only warmed
fluid should be used to avoid hypothermia. Excessive analgesia should be avoided in the acute stages of burn
resuscitation, as such medications mask signs of hypoxia and hypotension, which are used to guide fluid therapy.

37. What is the commonest type of fibroid?


a. Cervical
b. Interstitial
c. Subserous
d. Submucosal

Ans: B, Interstitial
Ref: Dutta’s Gynecology, 6th revised edition, page 272

21
ONLINE PG NEPAL QAE Magh 1

38. What is the lower limit of hemoglobin in physiologic anemia of pregnancy?


a. 12 gm%
b. 11 gm%
c. 10 gm%
d. 9 gm%

Ans: C, 10 gm%
Ref: Dutta’s Obstetrics, 8th edition, page 304
Criteria of physiological anemia: The lower limit of physiological anemia during the second half of pregnancy should fulfill
the following hematological values:
o Hb-10 gm%
o RBC-3.2 million/mm3
o PCV-32%
o Peripheral smear showing normal morphology of the RBC with central pallor.

39. Which of the following options best defines a cyclical bleeding of 100 ml occurring at normal interval?
a. Normal menses
b. Menorrhagia
c. Metrorrhagia
d. Polymenorrhea

Ans: B, Menorrhagia
Ref: Dutta’s Gynecology, 6th revised edition, page 185 – 186
 Menorrhagia is defined as cyclic bleeding at normal intervals; the bleeding is either excessive in amount (>
80 mL) or duration (>7 days) or both.
 Polymenorrhea is defined as cyclic bleeding where the cycle is reduced to an arbitrary limit of less than 21
days and remains constant at that frequency. If the frequent cycle is associated with excessive and or
prolonged bleeding, it is called epimenorrhagia.
 Metrorrhagia is defined as irregular, acyclic bleeding from the uterus. Amount of bleeding is variable.

22
ONLINE PG NEPAL QAE Magh 1

 Menstrual bleeding occurring more than 35 days apart and which remains constant at that frequency is
called oligomenorrhea.
 When the menstrual bleeding is unduly scanty and lasts for less than 2 days, it is called hypomenorrhea.

40. What is the commonest cause of female factor infertility?


a. Ovarian factor
b. Peritoneal factor
c. Tubal factor
d. Uterine factor

Ans: A, Ovarian factor


Ref: Dutta’s Gynecology, 6th revised edition, page 229
 Causes of female infertility: According to FIGO manual (1990) causes are: Tubal and peritoneal factors (25–
35%), Ovulatory factor (30–40%) and Endometriosis (1–10%).
 Ovarian factors: The ovulatory dysfunctions (dysovulatory) encompass:
o Anovulation or oligo-ovulation
o Decreased ovarian reserve
o Luteal phase defect (LPD)
o Luteinized unruptured follicle (LUF).

41. Pregnancy-associated plasma protein—A is secreted from which of the following structures?
a. Adrenals of fetus
b. Cytotrophoblast
c. Syncytiotrophoblast
d. Ovaries

Ans: C, Syncytiotrophoblast
Ref: Dutta’s Obstetrics, 8th edition, page 68
Pregnancy-associated plasma protein—A (PAPP-A) is secreted by the syncytiotrophoblast. It acts as an
immunosuppressant in pregnancy.

42. Which of the following is not a feature of placenta previa?


a. Bright red blood
b. Fetal heart sound usually present
c. Recurrent, painless bleeding
d. Uterus height disproportionately enlarged

Ans: D, Uterus height disproportionately enlarged


Ref: Dutta’s Obstetrics, 8th edition, page 287

23
ONLINE PG NEPAL QAE Magh 1

43. What is not an indication of expectant management of unruptured tubal ectopic pregnancy?
a. Fetal heart beat present on transvaginal sonography
b. Gestation sac size less than 4 cm
c. Initial serum hCG level less than 1,000 IU/L
d. No evidence of bleeding on transvaginal sonography

Ans: A, Fetal heart beat present on transvaginal sonography


Ref: Dutta’s Obstetrics, 8th edition, page 216
 Indication of expectant management of unruptured tubal pregnancy are:
o Initial serum hCG level less than 1,000 IU/L and the subsequent levels are falling.
o Gestation sac size less than 4 cm.
o No fetal heart beat on TVS.
o No evidence of bleeding or rupture on TVS

44. What is false about Braxton Hicks contractions?


a. Associated with simultaneous hardening of the uterus
b. Occurs at the onset of labor
c. Presence of abdominal pain
d. Pain radiates to thighs

Ans: B, Occurs at the onset of labor


Ref: Dutta’s Obstetrics, 8th edition, page 137

24
ONLINE PG NEPAL QAE Magh 1

Throughout pregnancy, painless Braxton Hicks contractions with simultaneous hardening of the uterus occur. These
contractions change their character, become more powerful, intermittent and are associated with pain. Pain more often
felt in front of the abdomen or radiating toward the thighs.

45. What is false about lactation and ovulation?


a. Lactation provides a natural method of contraception
b. In non lactating mothers ovulation occurs after 10 weeks
c. Duration of anovulation depends upon the frequency, intensity and duration of breastfeeding
d. It is possible for the patient to become pregnant before she menstruates

Ans: B, In non lactating mothers ovulation occurs after 10 weeks


Ref: Dutta’s Obstetrics, 8th edition, page 171 – 172
 In non lactating mothers, ovulation may occur as early as 4 weeks and in lactating mothers about 10
weeks after delivery. Duration of anovulation depends upon the frequency (>8/24 hours), intensity and
duration of breastfeeding. The physiological basis of anovulation and amenorrhea is due to elevated levels
of serum prolactin associated with suckling.
 In lactating mothers the mechanism of amenorrhea and anovulation are depicted schematically below.
Women who is exclusively breastfeeding, the contraceptive protection is about 98% up to 6 months of
postpartum.
 Thus, lactation provides a natural method of contraception. However, ovulation may precede the first
menstrual period in about one-third and it is possible for the patient to become pregnant before she
menstruates following her confinement. Non lactating mother should use contraceptive measures in 3rd
postpartum week and the lactating mother in 3rd postpartum month.

46. What is the commonest cause of fourth degree perineal tear?


a. Episiotomy
b. Large baby
c. Mismanaged second stage of labor
d. Primigravid mother

Ans: C, Mismanaged second stage of labor


Ref: Dutta’s Obstetrics, 8th edition, page 489
 While minor injury is quite common especially during first birth, gross injury (third and fourth degree) is
invariably a result of mismanaged second stage of labor.
 Overall risk is 1% of all vaginal deliveries.
 Causes: Perineal injury (mainly the third and fourth degree) results from (i) over stretching and/or (ii) rapid
stretching of the perineum especially when the perineum is inelastic (elderly primigravida, perineal scar).
 Prevention: Proper conduct in the second stage of labor taking due care of the perineum when it is likely
to be damaged is essential.

47. At least what oral temperature should be measured on two separate occasions a day apart to label a case as
puerperal pyrexia?
a. 99°F
b. 100°F
c. 100.4°F

25
ONLINE PG NEPAL QAE Magh 1

d. 101.2°F

Ans: C, 100.4°F
Ref: Dutta’s Obstetrics, 8th edition, page 500
A rise of temperature reaching 100.4°F (38°C) or more (measured orally) on two separate occasions at 24 hours apart
(excluding first 24 hours) within first 10 days following delivery is called puerperal pyrexia.

48. AFP is a tumor marker of which of the following cancer?


a. Dysgerminoma
b. Endodermal sinus tumor
c. Granulosa cell tumor
d. Ovarian epithelial carcinoma

Ans: B, Endodermal sinus tumor


Ref: Dutta’s Gynecology, 6th revised edition, page 521

49. A 14-year-old patient from very rural area comes to see you complaining of increasing fatigue and breathlessness on
exertion. On examination she appears cyanosed and has bilateral basal fine crepitations and a soft pansystolic murmur
with a displaced apex beat. She has never been in hospital and has no surgical scars. You urgently refer her for a
cardiology review. What is the most likely diagnosis?
a. Bacterial endocarditis
b. Tetralogy of Fallot
c. VSD producing a left-to-right shunt
d. Eisenmenger’s syndrome

26
ONLINE PG NEPAL QAE Magh 1

Ans: d
Ref: OP Ghai Pediatrics 9th Edition, Page No: 425
 This child grew up in an area without resources for congenital cardiac corrective surgery. She likely has a large VSD
which has led to left ventricular hypertrophy and failure, to the extent that she now has pulmonary hypertension
and a right-to-left shunt resulting in cyanosis, i.e. Eisenmenger’s syndrome.
 There are no infective features to suggest bacterial endocarditis.
 Without surgical correction, a child with Tetralogy of Fallot would not survive to their teens.
 Young children with a VSD have a left-to-right shunt with blood flowing from the high pressure left ventricle to the
right ventricle and this does not produce cyanosis.

50. A 6-year-old girl presents to hospital with a large right-sided abdominal mass. It does not cross the midline. On
further questioning she has had macroscopic haematuria and weight loss of 4 kg over the last 4 months. She has
reduced appetite and lethargy. Her blood pressure is 125/73 mmHg, heart rate 120 bpm. Which of the following is not a
complication of this malignancy?
a. Malnutrition
b. Hypertension
c. Renal impairment
d. Urinary catecholamines

Ans: d
Ref: OP Ghai Pediatrics 9th Edition, Page No: 609, 610
Wilms’ tumour can result in hypertension and renal impairment, dependent on the functioning of the contralateral kidney.
General complications of malignant disease include metastases and malnutrition due to poor appetite, vomiting and
increased metabolic demands.
Sympathetic nervous system stimulation does not occur with Wilms’ tumours but with neuroblastoma, an important
differential diagnosis, due to catecholamine production from the tumour which originates from the adrenal medulla.
Catecholamines can be detected in urine samples.

51. Which of the following is common endocrinological feature of Langerhans cell histiocytosis?
a. Hypothyroidism

27
ONLINE PG NEPAL QAE Magh 1

b. Diabetes insipidus
c. Pancytopenia
d. Pheochromocytoma

Ans: b
Ref: OP Ghai Pediatrics 9th Edition, Page No: 615

Thyroid gland involvement by Langerhans cell histiocytosis is extremely rare.

52. Invasive aspergillosis can have mortality rate upto:


a. 1 %
b. 10 %
c. 50 %
d. 100 %

Ans: c
Ref: OP Ghai Pediatrics 9th Edition, Page No: 253
Aspergillus is a ubiquitously distributed filamented fungus; the two common species causing human infection are A.
fumigatus and A. niger. Aspergillus causes certain non-invasive infections like otomycosis, sinusitis, aspergilloma and
allergic bronchopulmonary aspergillosis.
More sinister is invasive aspergillosis which can have mortality as high as 50%.

53. Total mOsm/L of WHO ORS is:


a. 322
b. 301
c. 285
d. 245

28
ONLINE PG NEPAL QAE Magh 1

Ans: d
Ref: OP Ghai Pediatrics 9th Edition, Page No: 290

54. During the first few days of life, the stool color in breastfed neonates changes by the end of first week as:
a. Green to Black
b. Black to Yellow
c. Brown to Black
d. Black to Green

Ans: b
Ref: OP Ghai Pediatrics 9th Edition, Page No: 136
Frequent stools:
 During the first few days of life, the stool color in breastfed neonates changes from black-green to yellow by the
end of first week. In between, the stools appear loose ('transitional stools').
 The stool frequency may increase at this time. It is attributed to the enhanced gastrocolic reflex which results in
the passage of small stools just after feeding.
 If the baby remains well hydrated, has no signs of sepsis, feeds well, passes urine 6-8 times per day and gains
weight, there is no cause for concern.

55. The adequacy of feeds in newborn can be determined by:


a. Passage of urine 2 to 3 times a day
b. Baby sleeping well for 2- 3 hours after feeds
c. Good cry
d. Normal capillary refill time

Ans: b
Ref: OP Ghai Pediatrics 9th Edition, Page No: 135
The adequacy of feeds can be determined by:
 Passage of urine 6 to 8 times every 24 hours
 Baby sleeping well for 2- 3 hours after feeds

56. Unidextrous reach is seen earliest by:


a. 4 months
b. 6 months

29
ONLINE PG NEPAL QAE Magh 1

c. 9 months
d. 12 months

Ans: b
Ref: OP Ghai Pediatrics 9th Edition, Page No: 46

57. Cold injury includes:


a. Frostnip
b. Immersion foot
c. Trench foot
d. All of the above

Ans: d
Ref: Nelson pediatrics, 20th Edition, Page No: 577
The involvement of children and youth in snowmobiling, mountain climbing, winter hiking, and skiing places them at risk
for cold injury.
Cold injury may produce either local tissue damage, with the injury pattern depending on exposure to damp cold (frostnip,
immersion foot, or trench foot), dry cold (which leads to local frostbite), or generalized systemic effects (hypothermia).
Hypothermia occurs when the body can no longer sustain normal core temperature by physiologic mechanisms, such as
vasoconstriction, shivering, muscle contraction, and nonshivering thermogenesis.
When shivering ceases, the body is unable to maintain its core temperature; when the body core temperature falls to
<35°C (95°F), the syndrome of hypothermia occurs. Wind chill, wet or inadequate clothing, and other factors increase local
injury and may cause dangerous hypothermia, even in the presence of an ambient temperature that is not <17-20°C (50-
60°F).

58. The goal of incubator care is to maintain the core temperature of infant at:
a. 36.6-37.5°C
b. 30.6-31.5°C
c. 39.6-40.5°C

30
ONLINE PG NEPAL QAE Magh 1

d. 42 °C

Ans: a
Ref: OP Ghai Pediatrics 9th Edition, Page No: 143, 144
Incubators and radiant wanners:
These equipment are used to assist sick and small neonates maintain their normal body temperature.
Incubator is a transparent acrylic cabin which has warm air circulating around the baby to keep him warm. There is an
inbuilt feedback system (servo-control) that controls ambient temperature inside incubator by altering heater output
based on baby's temperature and thereby maintains the temperature of baby in the normal range.
A radiant warmer is an open system (as compared to incubator which is a closed cabinet) and the neonate lies on a crib.
There is overhead radiant warmer that modulates its heater output based on baby's temperature sensed by a skin probe.
Radiant warmers and incubators should be used m the servo-control mode with the abdominal skin temperature
maintained at 36.5°C to 37°C.

59. Pott's shunt is


a. Right subclavian artery to right pulmonary artery
b. Descending aorta to left pulmonary artery
c. Left subclavian to left pulmonary artery
d. Ascending aorta to right pulmonary artery

Ans: b
Ref: Potts Shunt in Patients with Primary Pulmonary Hypertension; Sue Hyun Kim; Korean J Thorac Cardiovasc Surg. 2015
Feb; 48(1): 52–54.
The Potts shunt, which creates an anastomosis between the left pulmonary artery and the descending aorta, has been
proposed as a theoretically promising palliative surgical technique to decompress the right ventricle.

60. Chloroma is seen in:


a. CML
b. AML
c. All
d. CLL

Ans: b
Ref: OP Ghai Pediatrics 9th Edition, Page No: 597
 The clinical presentation of AML is similar to ALL but more likely to have high TLC and incidence of infections.
 Unlike ALL, lymphadenopathy and massive hepatosplenomegaly is not very common.
 However, infants and toddlers with M4 and MS AML subtypes have more organomegaly, high leukocyte counts
and CNS disease at diagnosis.
 Gum hypertrophy a common feature of the M4 subtype.
 Disseminated intravascular coagulation may occur with any subgroup, but is common in acute promyelocytic
leukemia (M3).
 Chloromas are localized collections of leukemic cells that which may occur at any site including CNS, neck, bones
(typically orbit) and skin.
 Patients with high TLC may present with signs of leukostasis such as pulmonary infiltrates causing respiratory
distress or stroke.

31
ONLINE PG NEPAL QAE Magh 1

 Central nervous system involvement may occur in up to 15% patients.

ENT
61. What is not a blood supply of adenoids?
a. Ascending cervical branch of inferior thyroid artery of thyrocervical trunk
b. Ascending palatine branch of facial
c. Descending pharyngeal branch of external carotid
d. Pharyngeal branch of the third part of maxillary artery

Ans: C, Descending pharyngeal branch of external carotid


Ref: Dhingra ENT, 7th edition, page 275
Adenoids receive their blood supply from:
 Ascending palatine branch of facial.
 Ascending pharyngeal branch of external carotid.
 Pharyngeal branch of the third part of maxillary artery.
 Ascending cervical branch of inferior thyroid artery of thyrocervical trunk.
Lymphatics from the adenoid drain into upper jugular nodes directly or indirectly via retropharyngeal and parapharyngeal
nodes.
Nerve supply is through CN IX and X. They carry sensation. Referred pain to ear due to adenoiditis is also mediated
through them.

62. What is the treatment of mucocele?


a. Deroofing
b. Electrocautery
c. Excision
d. Silver nitrate

Ans: C, Excision
Ref: Dhingra ENT, 7th edition, page 252
Mucocele: Most common site is the lower lip. It is a retention cyst of minor salivary glands of the lip. The lesion appears as
a soft and cystic mass of bluish colour. Treatment is surgical excision.

63. Which of the following muscle doesn’t elevate the larynx?


a. Palatopharyngeus
b. Salpingopharyngeus
c. Sternohyoid
d. Thyrohyoid

Ans: C, Sternohyoid
Ref: Dhingra ENT, 7th edition, page 321
Extrinsic muscles. They connect the larynx to the neighbouring structures and are divided into elevators or depressors of
larynx.
(a) Elevators.
 Primary elevators act directly as they are attached to the thyroid cartilage and include stylopharyngeus,
salpingopharyngeus, palatopharyngeus and thyrohyoid.

32
ONLINE PG NEPAL QAE Magh 1

 Secondary elevators act indirectly as they are attached to the hyoid bone and include mylohyoid (main), digastric,
stylohyoid and geniohyoid.
(b) Depressors. They include sternohyoid, sternothyroid and omohyoid.

64. Which of the following is not a congenital cause of stridor?


a. Hemangioma
b. Laryngeal web
c. Papillomatosis
d. Subglottic stenosis

Ans: C, Papillomatosis
Ref: Dhingra ENT, 7th edition, page 334

65. What is preserved in type I modified radical neck dissection?


a. Cranial nerve XI
b. Cranial nerve XI and internal jugular vein
c. Cranial nerve XI, internal jugular vein and sternocleidomastoid
d. Cranial nerve XI, internal jugular vein, sternocleidomastoid and supraomohoid

Ans: A, Cranial nerve XI


Ref: Dhingra ENT, 7th edition, page 442
Classification of neck dissection
 1. Radical neck dissection.
 2. Modified radical neck dissection.
o (a) Type I—Preserves CN XI
o (b) Type II—Preserves CN XI and internal jugular vein
o (c) Type III—Preserves CN XI, internal jugular vein and sternocleidomastoid muscle.
 3. Selective neck dissection
o (a) Supraomohyoid (or anterolateral) (removes level I to III)
o (b) Lateral (removes nodes in level II, III, IV)

33
ONLINE PG NEPAL QAE Magh 1

o (c) Posterolateral (removes level II to V suboccipital and postauricular nodes)


o (d) Anterior compartment (removes level VI nodes)
 4. Extended neck dissection (vide infra).

66. While investigating the cause of cavernous sinus thrombosis, you find that supraorbital veinis the route of the
problem. What is the likely source?
a. Ethmoid sinus
b. Frontal sinus
c. Orbit
d. Sphenoid sinus

Ans: B, Frontal sinus


Ref: Dhingra ENT, 7th edition, page 226

67. What does presence of Holman-Miller sign in CT scan of head reveal?


a. Angiofibroma
b. Choanal polyp
c. Epignathi
d. Thornwaldt’s cyst

Ans: A, Angiofibroma
Ref: Dhingra ENT, 7th edition, page 281
 Computed tomography (CT) scan of the head with contrast enhancement is now the investigation of choice. It has
replaced conventional radiographs.

34
ONLINE PG NEPAL QAE Magh 1

 It shows the extent of tumour, bony destruction or displacements. Anterior bowing of the posterior wall of
maxillary sinus, often called antral sign or Holman-Miller sign, is pathognomic of angiofibroma.

68. What is not a complication of otitis media?


a. Brain abscess
b. Cholesteatoma formation
c. Lateral sinus thrombophlebitis
d. Petrositis

Ans: B, Cholesteatoma formation


Ref: ENT Dhingra, 7th edition, page 83 – 84
Sequelae of otitis media are the direct result of middle ear infection and should be differentiated from complications. They
include:
o Perforation of tympanic membrane
o Ossicular erosion
o Atelectasis and adhesive otitis media
o Tympanosclerosis
o Cholesteatoma formation
o Conductive hearing loss due to ossicular erosion or fixation
o Sensorineural hearing loss
o Speech impairment
o Learning disabilities

The last two are secondary to loss of hearing in the developmental phase of the infant or child.
Complications of otitis media are classified into two main groups:
A. Intratemporal (within the confines of temporal bone)
 1. Mastoiditis
 2. Petrositis
 3. Facial paralysis
 4. Labyrinthitis
B. Intracranial
 1. Extradural abscess
 2. Subdural abscess
 3. Meningitis
 4. Brain abscess
 5. Lateral sinus thrombophlebitis
 6. Otitic hydrocephalus.

69. A chalky plaque tympanic membrane seen on inspection will likely reveal which underlying condition?
a. Acute otitis media
b. Hemotympanum
c. Secretory otitis media
d. Tympanosclerosis

Ans: D, Tympanosclerosis

35
ONLINE PG NEPAL QAE Magh 1

Ref: Dhingra ENT, 7th edition, page 429 – 430


Normal tympanic membrane is pearly white in colour and semitransparent and obliquely set at the medial end of the
meatus. It has two parts—pars tensa and pars flaccida, both of which should be carefully examined. A tympanic
membrane is examined for:
o Colour. Red and congested in acute otitis media, bluish in secretory otitis media or haemotympanum. A chalky
plaque is seen in tympanosclerosis.
o Position. Tympanic membrane may be retracting or bulging. General retraction is seen in tubal occlusion,
retraction pockets are seen in attic or posterosuperior region and may collect epithelial flakes. Warming a
laryngeal mirror over a spirit lamp. Warm only the glass side of the mirror. tympanic membrane is very thin,
deeply retracted and is fixed to promontory as in adhesive otitis media. Bulging tympanic membrane is seen in
acute otitis media, haemotympanum or neoplasm of middle ear which has not yet perforated the drum.
o Surface of tympanic membrane. It may show vesicles or bullae (herpes zoster or myringitis bullosa), a perforation
(acute or chronic otitis media). A perforation may be central (in pars tensa) or attic (in pars flaccida) or marginal
(at the periphery involving the annulus). A central perforation may be small, medium, subtotal or total.
o Mobility. It is tested with a Siegle’s speculum. A normal tympanic membrane is mobile. Restricted mobility is seen
in the presence of fluid or adhesions in the middle ear. An atrophic segment of tympanic membrane may be
hypermobile.

70. A person with hearing defect underwent tympanography. The finding showed that maximum compliance
occurred with negative pressure in excess of 100 mm H 2O. What is the likely cause?
a. Fixation of ossicles
b. Normal finding
c. Ossicular discontinuity
d. Retracted tympanic membrane

Ans: D, Retracted tympanic membrane


Ref: Dhingra ENT, 7th edition, page 27

Type A Normal tympanogram.

36
ONLINE PG NEPAL QAE Magh 1

o Type As Compliance is lower at or near ambient air pressure. Seen in fixation of ossicles, e.g. otosclerosis or
malleus fixation.
o Type Ad High compliance at or near ambient pressure. Seen in ossicular discontinuity or thin and lax tympanic
membrane.
o Type B A flat or dome-shaped graph. No change in compliance with pressure changes. Seen in middle ear fluid or
thick tympanic membrane.
o Type C Maximum compliance occurs with negative pressure in excess of 100 mm H2O. Seen in retracted tympanic
membrane and may show some fluid in middle ear.

Orthopedics
71. In CTEV, all of the following deformities are seen except:
a. Forefoot-adduction
b. Equinus at ankle
c. Vertical talus
d. Varus at heel

Ans: c
Ref: Maheshwari Orthopedics 5th Edition Page No: 211
Congenital talipes equinovarus (CTEV) is considered the most common anomaly affecting the feet diagnosed on antenatal
ultrasound.
Talipes equinovarus consists of four elements:
 Hindfoot equinus: lateral talocalcaneal angle less than 35º
 Hindfoot varus: talocalcaneal angle less than 20º
 Metatarsus adductus: adduction and varus deformity of the forefoot; talus to first metatarsal angle greater than
15º
 Talonavicular subluxation: medial subluxation of the navicular on the talus

72. The Milwaukee brace is used for correction of:


a. Congenital dislocation of the hip
b. Club foot
c. Scoliosis
d. Patella alta

Ans: c
Ref: Maheshwari Orthopedics 5th Edition Page No: 282
 The Milwaukee brace is an active corrective spinal orthosis used almost exclusively in the ambulant treatment of
structural scoliosis the aim being to postpone temporarily or permanently the need for operation.
 It frequently has to be worn for a number of years until the spine is stable.

73. 'Corduroy' appearance on X-ray is a characteristic feature of (on X-ray for the spine):
a. Histiocytosis
b. Hemangioma
c. Synovial sarcoma
d. Osteomyelitis

37
ONLINE PG NEPAL QAE Magh 1

Ans: b
Ref: Apley Orthopedics; 9th Edition, Page No: 204
Osseous hemangiomas consist of vascular channels and are usually seen in middle aged patients, the spine being the
commonest site, radiologically the spine shows coarse vertical trabeculation this is called corduroy appearance.

74. Lisfranc’s dislocation involves:


a. Hand
b. Foot
c. Spine
d. Pelvis

Ans: b
Ref: Maheshwari Orthopedics 5th Edition Page No: 367
Dislocations with eponyms
 Chopart’s: dislocation through talo-navicular joints
 Divergent: elbow dislocation where ulna and radius dislocate in opposite directions
 Lisfranc’s: dislocation through inter tarsal joint
 Lunate: wrist injury where lunate bone comes out to lie in front of other carpal bones
 Luxatio erecta: inferior dislocation of shoulder
 Otto pelvis: gradual shift of the acetabulum into the pelvis (e.g. in osteomalacia)
 Perilunate: wrist injury where the lunate remains in its place and the other carpal bones dislocate around it
dorsally
 Spondylolisthesis: movement of one vertebra over another (usually L4 over L5)

75. What is the mechanism of action of capacitive coupling (CC) stimulation when used as an adjunctive therapy for
bone healing?
a. Reduces oxygen concentration and increases local tissue pH
b. Stimulates transmembrane calcium translocation via voltage-gated calcium channels
c. Upregulates TNF alpha
d. Transmits mechanical energy to stimulate bone formation

Ans: b
 Adjunctive therapies for bone healing are widely used and the mechanism of action is slowly being elucidated.
Capacitive coupling involves externally placed electrodes with an alternating current which creates an electrical
field. This stimulates calcium translocation which then activates calmodulin and upregulates many factors involved
in bone healing (BMP, cyclic adenosine monophosphate (cAMP), and TGF-beta1).
 Direct current (DC) stimulates an inflammatory-like response during fracture repair while . Pulsed electromagnetic
fields (PEMFs) cause calcification of fibrocartilage but not calcification of fibrous tissue.

76. In flexion deformity of hip, patient can walk straight by:


a. Compensatory lumbar lordosis
b. Compensatory dorsal lordosis
c. Compensatory sacral lordosis
d. Added pelvic tilt

38
ONLINE PG NEPAL QAE Magh 1

Ans: a
Ref: Maheshwari Orthopedics 5th Edition Page No: 348
In flexion deformity:
 With mild flexion deformity of the hip, the patient manages to walk ‘straight’ by compensatory lumbar lordosis.
 If the deformity is more than 30o, the patient can no longer compensate, and is required to stoop forward at the
hip to be able to walk.
 This also happens in patients with ankylosing spondylitis, where compensatory lumbar lordosis is not possible due
to stiffness of the spine.

77. Cervical rib arises from:


a. 4th cervical vertebra
b. 5th cervical vertebra
c. 6th cervical vertebra
d. 7th cervical vertebra

Ans: d
Ref: Maheshwari Orthopedics 5th Edition Page No: 327
Cervical rib arises from 7th cervical vertebra.

78. Rachitic rosary means:


a. Skull gives the feeling of a ping pong ball
b. Broadening of the ends of long bones
c. Horizontal depression, along the lower part of the chest
d. Costochondral junctions on the anterior chest wall become prominent

Ans: d
Ref: Maheshwari Orthopedics 5th Edition Page No: 310, 311
Clinical features
Nutritional rickets occurs in children about 1 year old.
It may occur in older children with mal-absorption syndrome. Following are the clinical features:
• Craniotabes: This is the manifestation of rickets seen in young infants. Pressure over the soft membranous bones
of the skull gives the feeling of a ping pong ball being compressed and released.
• Bossing of the skull: Bossing of the frontal and parietal bones becomes evident after the age of 6 months.
 Broadening of the ends of long bones, most prominently around wrists and knees. It is seen around 6-9 months of
age.
• Delayed teeth eruption is noticed in infants.
• Harrison's sulcus: A horizontal depression, along the lower part of the chest, corresponding to the insertion of
diaphragm.
• Pigeon chest: The sternum is prominent.
• Rachitic rosary: The costochondral junctions on the anterior chest wall become prominent, giving rise to
appearance of a rosary.
• Muscular hypotonia: The child's abdomen becomes protruberant (pot belly) because of marked muscular
hypotonia. Visceroptosis and lumbar lordosis occurs.
• Deformities: Deformities of the long bones resulting in knock knees or bow legs is a common presentation of
rickets, once the child starts walking.

39
ONLINE PG NEPAL QAE Magh 1

79. Number of affected vertebrae in knuckle kyphus:


a. 1
b. 2-3
c. 3-4
d. 4-6

Ans: a
Ref: Maheshwari Orthopedics 5th Edition Page No: 189
Number of affected vertebrae
• Knuckle 1
• Gibbus 2-3
• Angular kyphosis 3-4
• Rounded kyphosis >4
In different references:
• Knuckle kyphus: Involving 1 vertebra
• Angular kyphus: Involving 2-3 vertebra
• Rounded kyphus: Involving more than 3 vertebra
(Note: Gibbus is an old term not used these days more or less equivalent to Angular Kyphus.)

80. Gunstock's deformity is seen following


a. Supracondylar fracture humerus
b. Rib Fracture
c. Trimalleloar Fracture ankle
d. Fracture spinous process C7

Ans: a
Ref: Ebnezar Orthopedics 4th Edition, Page No: 155
Cubitus varus (Gunstock elbow) is called so because the deformity resembles a rifle gunstock.
This is the most common complication of supracondylar fracture.
The deformity becomes obvious in an extended elbow.
The following are three static deformities of cubitus varus (all with respect to distal fragment):
• Posterior displacement
• Horizontal rotation
• Coronal tilt
Causes of cubitus varus: 4 ‘I’s
• Improper persons treating
• Improper reduction
• Improper interpretation of radiographs
• Improper follow-up

Ophthalmology
81. What type of blindness does a person with vision in better eye <3/60 to 1/60 have?
a. Economic blindness
b. Legal blindness

40
ONLINE PG NEPAL QAE Magh 1

c. Social blindness
d. Total blindness

Ans: C, Social Blindness


Ref: Khurana Ophthalmology, 6th edition, page 475 – 476
 WHO definition of blindness. In order to have comparable national and international statistics, the WHO in
1972 proposed a uniform criterion and defined blindness as, “Visual acuity of less than 3/60 (Snellen) or
its equivalent”. In order to facilitate the screening of visual acuity by nonspecialised persons, in the
absence of appropriate vision charts, WHO in 1979 added the “Inability to count fingers in daylight at a
distance of 3 metres” to indicate vision less than 3/60 or its equivalent.3 Visual field less than 10°,
irrespective of the level of visual acuity is also labelled as blindness (WHO, 1977).
 Other definitions of blindness are:
o Economic blindness: Vision in better eye <6/60 to 3/60
o Social blindness: Vision in better eye <3/60 to 1/60.
o Legal blindness: Vision in better eye <1/60 to perception light
o Total blindness: No light perception (PL-ve).

82. The nasolacrimal duct lies in a bony canal formed by which of the following?
a. Maxilla and inferior turbinate
b. Maxilla and medial turbinate
c. Nasal and inferior turbinate
d. Nasal and medial turbinate

Ans: A, Maxilla and inferior turbinate


Ref: Khurana Ophthalmology, 6th edition, page 387
Nasolacrimal duct (NLD)
 Dimensions and location. It extends from neck of the lacrimal sac to inferior meatus of the nose. It is
about 15–18 mm long and lies in a bony canal formed by the maxilla and the inferior turbinate.
 Direction of the NLD is downwards, backwards and laterally. Externally, its location is represented by a line
joining inner canthus to the ala of nose. The upper end of the NLD is the narrowest part. Numerous
membranous valves are present in the NLD, the most important is the valve of Hasner, which is present at
the lower end of the duct and prevents reflux from the nose.

83. What is a closed-globe injury?


a. Double perforation
b. Intraocular foreign body
c. Lamellar laceration
d. Rupture

Ans: C, Lamellar laceration


Ref: Khurana Ophthalmology, 6th edition, page 426
 The definitions proposed by the BETT and as such adopted by ‘American Ocular Trauma Society’ (AOTS)
for mechanical ocular injuries are as follows:
 1. Closed-globe injury is the one in which eye wall (sclera and cornea) does not have a full thickness
wound but there is intraocular damage. It includes contusion and lamellar laceration.

41
ONLINE PG NEPAL QAE Magh 1

o Contusion refers to the closed-globe injury resulting from blunt trauma. Damage may occur at the
site of impact or at a distant site.
o Lamellar laceration is a closed-globe injury characterized by a partial thickness wound of the
eyewall caused by a sharp object or blunt trauma.
 2. Open-globe injury is associated with a full-thickness wound of the sclera or cornea or both. It includes
rupture and laceration of eyewall.
o Rupture refers to a full-thickness wound of eyewall caused by the impact of blunt trauma. The
wound occurs due to markedly raised intraocular pressure by an inside-out injury mechanism.
o Laceration refers to a full-thickness wound of eyewall caused by a sharp object. The wound occurs
at the impact site by an outside-in mechanism. It includes penetrating and perforating injuries.
 Penetrating injury refers to a single laceration of eyewall caused by a sharp object which
traverses the coats only once.
 Perforating injury refers to two full thickness lacerations (one entry and one exit) of the
eyewall caused by a sharp object or missile. The two wounds must have been caused by
the same agent (earlier known as double perforation).
 lntraocular foreign body injury is technically a penetrating injury associated with retained
intraocular foreign body. However, it is grouped separately because of different clinical
implications.

84. Which of the following represents a congenital nuclear cataract?


a. Anterior polar cataract
b. Blue dot cataract
c. Congenital membranous cataract
d. Floriform cataract

Ans: D, Floriform cataract


Ref: Khurana Ophthalmology, 6th edition, page 183
Congenital and developmental cataracts have been variously classified. A simple morphological classification of congenital
and developmental cataract is as under:
 I. Congenital capsular cataracts
o 1. Anterior capsular cataract
o 2. Posterior capsular cataract
 II. Polar cataracts
o 1. Anterior polar cataract
o 2. Posterior polar cataract
 III. Congenital nuclear cataracts
o 1. Cataracta pulverulenta
o 2. Lamellar cataract
o 3. Sutural and axial cataracts
 Floriform cataract
 Coralliform cataract
 Spear-shaped cataract
 Anterior axial embryonic cataract
 Dendritic suture cataract
o 4. Total nuclear cataract

42
ONLINE PG NEPAL QAE Magh 1

 IV. Generalized cataracts


o 1. Coronary cataract
o 2. Blue dot cataract
o 3. Total congenital cataract
o 4. Congenital membranous cataract

85. Open-sky technique is employed to perform which of the following ocular surgeries?
a. Anterior vitrectomy
b. Closed vitrectomy
c. Posterior vitrectomy
d. Subtotal vitrectomy

Ans: A, Anterior vitrectomy


Ref: Khurana Ophthalmology, 6th edition, page 261
Vitrectomy is the surgical removal of the vitreous. Common terms are:
o Anterior vitrectomy: It refers to removal of anterior part of the vitreous.
o Core vitrectomy: It refers to removal of the central bulk of the vitreous. It is usually indicated in
endophthalmitis.
o Subtotal & total vitrectomy: In this almost whole of the vitreous is removed.
o Open-sky vitrectomy technique is employed to perform only anterior vitrectomy.
o Para plana approach (closed vitrectomy) is employed to perform anterior vitrectomy, core vitrectomy,
subtotal and total vitrectomy.

86. In which type of asymmetrical aniseikonia does the visual distortion decrease progressively in both directions?
a. Barrel distortion
b. Pincushion distortion
c. Prismatic distortion
d. Oblique distortion

Ans: A, Barrel distortion


Ref: Khurana Ophthalmology, 6th edition, page 45
Aniseikonia is defined as a condition wherein the images projected to the visual cortex from the two retinae are
abnormally unequal in size and/or shape. Clinically, aniseikonia may be of different types:
 Symmetrical aniseikonia
o a. Spherical, image may be magnified or minified equally in both meridians.
o b. Cylindrical, image is magnified or minified symmetrically in one meridian.
 Asymmetrical aniseikonia
o a. Prismatic. In this image, difference increases progressively in one direction.
o b. Pincushion. In this image, distortion increases progressively in both directions, as seen with
high plus correction in aphakia.
o c. Barrel distortion. In this image, distortion decreases progressively in both directions, as seen
with high minus correction.
o d. Oblique distortion. In this, the size of image is same, but there occurs an oblique distortion
of shape.

43
ONLINE PG NEPAL QAE Magh 1

87. Which of the following anti-glaucoma drug acts as a dual action parasympathomimetic agent?
a. Atropine
b. Carbachol
c. Demecarium
d. Pilocarpine

Ans: B, Carbachol
Ref: Khurana Ophthalmology, 6th edition, page 449
 Parasympathomimetics, also called as cholinergic drugs, either imitate or potentiate the effects of
acetylcholine. Depending upon the mode of action, these can be classified as follows:
o Direct-acting or agonists, e.g., pilocarpine.
o Indirect-acting parasympathomimetics or cholinesterase inhibitors: As the name indicates these
drugs act indirectly by destroying the enzyme cholinesterase; thereby sparing the naturally acting
acetylcholine for its actions. These drugs have been divided into two subgroups, designated as
reversible (e.g., physostigmine) and irreversible (e.g., echothiophate iodide, demecarium and
diisopropyl fluorophosphate, DFP3) anti-cholinesterases.
o Dual-action parasympathomimetics, i.e., which act as both a muscarinic agonist as well as a weak
cholinesterase inhibitor, e.g., carbachol.

Anesthesiology
88. What is the most common postoperative complication of spinal anesthesia?
a. Bradycardia
b. Hypotension
c. Post-spinal headache
d. Urinary retention

Ans: D, Urinary retention


Ref: Ajay Yadav Anesthesia, 6th edition, page 171
 Blockade of sacral parasympathetic fibers (S2, 3, 4) is responsible for causing urinary retention. Due to
significant decrease in postspinal headache urinary retention has now become the most common
postoperative complication of spinal anesthesia.
 It can occur in as much as one-third of the patients. Concurrent administration of intrathecal opioids
especially morphine is strongly associated with higher incidence of urinary retention. Catheterization may
be required.

89. What is the level of block required for cesarean section?


a. T4
b. T6
c. T8
d. T10

Ans: A, T4
Ref: Ajay Yadav Anesthesia, 6th edition, page 179

44
ONLINE PG NEPAL QAE Magh 1

 When deciding the level of b lock it is important to block nerve supply of all the organs involved during
surgery, not only the level of skin incision. For example, skin incision level for cesarean section is at Tl1
level but since intestine and peritoneum are also handled, block required is up to T4.
 Level of block required for some common surgeries:
o Cesarean section Up to T4.
o Prostate Up to no.
o Testicular surgeries Up to TIO.
o Hernia Up to no.
o Appendix Up to T8.
o Hysterectomies Up to T6.
o Perianal surgeries Sacral segments.

90. Combination of which of the following ECG leads can detect majority of intraoperative ischemic events?
a. I and V2
b. I and V5
c. II and V2
d. II and V5

Ans: D, II and V5
Ref: Ajay Yadav Anesthesia, 6th edition, page 183
ECG is mandatory for intraoperative monitoring. Lead V5 is the best to diagnose left ventricle infarction and lead II for
arrhythmia (as well as inferior wall MI) therefore a combination of lead II and VS can detect majority (94-95%) of
intraoperative ischemic events.

91. What is false about ketamine?


a. Antagonist at NMDA receptor
b. Decreases intracranial pressure
c. Excreted in urine
d. Inhibits vagal nerve

Ans: B, Decreases intracranial pressure


Ref: Miller’s Anesthesia, 8th edition, page 845 – 850
 Ketamine produces dissociative anesthesia, rather than generalized depression of the CNS, through
antagonistic actions at the phencyclidine (PCP) site of the N-methyl-D-aspartate receptor (NMDAR).
 It is metabolized by hepatic microsomal enzymes. The major pathway involves N-demethylation to form
norketamine (metabolite I), which is then hydroxylated to hydroxynorketamine, which is further
conjugated to water-soluble glucuronide derivatives and excreted in the urine.
 Ketamine increases cerebral metabolism, cerebral blood flow, and intracranial pressure. Because of its
excitatory CNS effects, which can be detected by generalized EEG development of theta wave activity and
by petit mal seizure-like activity in the hippocampus, ketamine increases CMRO2.
 Ketamine, like other phencyclidines, produces undesirable psychological reactions, which occur during
awakening from ketamine anesthesia and are termed emergence reactions. The common manifestations
of these reactions, which vary in severity and classification, are vivid dreaming, extracorporeal experiences
(sense of floating out of body), and illusions (misinterpretation of a real, external sensory experience).

45
ONLINE PG NEPAL QAE Magh 1

 Ketamine increases arterial blood pressure, heart rate, and cardiac output in a biphasic manner. Ketamine
causes the systemic release of catecholamines, inhibition of the vagal nerve, inhibition of norepinephrine
reuptake at peripheral nerves and nonneuronal tissues such as the myocardium, and norepinephrine
release from sympathetic ganglia.

92. What is the inhalational agent of choice for maintenance of anesthesia for asthmatics?
a. Desflurane
b. Halothane
c. Isoflurane
d. Sevoflurane

Ans: D, Sevoflurane
Ref: Ajay Yadav Anesthesia, 6th edition, page 193
 Sevoflurane is the inhalational agent of choice for asthma patients. Halothane, in spite of producing little
more bronchodilatation than sevoflurane in asthma patients is not preferred because of the increase
possibility of arrhythmias.
 As desflurane and isoflurane have irritating effects on airways therefore should be avoided.

93. Which among the following benzodiazepine will you use if you want a longer duration of sedation?
a. Diazepam
b. Lorazepam
c. Midazolam
d. Temazepam

Ans: A, Diazepam
Ref: Miller’s Anesthesia, 8th edition, page 838
 The four benzodiazepines used in anesthesia are classified as a short-acting agent (midazolam),
intermediate-acting agents (lorazepam, temazepam), and a long-acting agent (diazepam), according to
their metabolism and plasma clearance.

94. What is the muscle relaxant of choice in a patient with coexistent renal ailment?
a. Cis-atracurium
b. Rocuronium
c. Succinyl choline
d. Vecuronium

Ans: A, Cis-atracurium
Ref: Ajay Yadav Anesthesia, 6th edition, page 203
 Muscle relaxant of choice for patients with renal disorders:
o As these patients exhibit hyperkalemia, succinylcholine should not be used.
o Due to Hoffman degradation non-depolarizing muscle relaxant of choice are Atracurium and Cis-
atracurium.

Psychiatry
95. Which of the following demonstrate impulsivity in a child with attention deficit hyperactivity disorder?

46
ONLINE PG NEPAL QAE Magh 1

a. Child has difficulty in waiting for turn at play


b. Child is easily distracted by external stimuli
c. Child moves here and there fidgeting
d. Child talks excessively

Ans: A, Child has difficulty in waiting for turn at play


Ref: Ahuja Psychiatry, 7th edition, page 166
 Attention deficit disorder with hyperactivity (Hyperkinetic disorder): This is the commonest type. The
characteristic clinical features are:
 Poor attention span with distractibility
o i. Fails to finish the things started.
o ii. Shifts from one uncompleted activity to another.
o iii. Doesn’t seem to listen.
o iv. Easily distracted by external stimuli.
o v. Often loses things.
 Hyperactivity
o i. Fidgety.
o ii. Difficulty in sitting still at one place for long.
o iii. Moving about here and there.
o iv. Talks excessively.
o v. Interference in other people’s activities.
 Impulsivity
o i. Acts before thinking, on the spur of the moment.
o ii. Difficulty in waiting for turn at work or play.

96. What test is used to assess adaptive behavior?


a. Binet-Simon test
b. Raven’s progressive matrices
c. Vineland Social Maturity Scale
d. Wechsler Intelligence Scale for Children

Ans: C, Vineland Social Maturity Scale


Ref: Ahuja Psychiatry, 7th edition, page 160
 The commonly used tests for measurement of intelligence include:
o i. Seguin form board test.
o ii. Stanford-Binet, Binet-Simon or Binet-Kamath tests.
o iii. Wechsler Intelligence Scale for Children (WISC) for 6½ to 16 years of age.
o iv. Wechsler’s Preschool and Primary Scale of Intelligence (WPPSI) for 4 to 6½ years of age.
o v. Bhatia’s battery of performance tests.
o vi. Raven’s progressive matrices (coloured, standard and advanced).
 The tests used for the assessment of adaptive behaviour include:
o i. Vineland Social Maturity Scale (VSMS).
o ii. Denver Development Screening Test (DDST).
o iii. Gessell’s Development Scale.

47
ONLINE PG NEPAL QAE Magh 1

97. Who was the proponent of cognitive behavior therapy?


a. Aaron T Beck
b. Donald Meichenbaum
c. Sigmund Freud
d. Wilhelm Reich

Ans: B, Donald Meichenbaum


Ref: Ahuja Psychiatry, 7th edition, page 213

98. A person is fully aware of being which he attributes to something unknown in self and yet does not apply this
knowledge to enhance his current experience. What is his level of insight?
a. 3
b. 4
c. 5
d. 6

Ans: B, 4
Ref: Ahuja Psychiatry, 7th edition, page 15

48
ONLINE PG NEPAL QAE Magh 1

99. Which of the following drug in adjunct to other modes of therapy is useful in inducing weight gain in a patient
with anorexia nervosa?
a. Amitriptyline
b. Clomipramine
c. Cyproheptadine
d. Fluoxetine

Ans: C, Cyproheptadine
Ref: Ahuja Psychiatry, 7th edition, page 144
 Drugs are an important adjunct to other modes of therapy for anorexia nervosa. The drugs used can
include:
 i. Antipsychotics: Chlorpromazine is rarely used these days. Olanzapine has efficacy in improving weight
gain but it is important to be aware of possibility of prolongation of QTc particularly in patients with low
BMI.
 ii. Antidepressants (such as fluoxetine, clomipramine) for treatment of anorexia nervosa and/or associated
depression.
 iii. Cyproheptadine: This is particularly helpful in inducing weight gain, decreasing depressive symptoms
and increasing appetite, if anorexia is actually present. The usual dose is 8-32 mg/day, in divided doses.
However, co-prescription with SSRIs can interfere with their effectiveness as Cyproheptadine is a
serotonin antagonist.

100. A person displaying apathy, lack of initiative, retardation and perseveration is likely to have which type of
personality disorder?
a. Bilateral temporal lobe
b. Frontal convexity type
c. Orbito-frontal syndrome
d. Medial frontal syndrome

Ans: B, Frontal convexity type


Ref: Ahuja Psychiatry, 7th edition, page 31

49
ONLINE PG NEPAL QAE Magh 1

101. What type of delusion is a patient suffering from schizophrenia having when she complaints of insects crawling
all over her?
a. Delusion of control
b. Delusion of grandeur
c. Hypochondriacal delusions
d. Persecutionary delusions

Ans: C, Hypochondriacal delusions


Ref: Ahuja Psychiatry, 7th edition, page 56
 The commonly seen delusions in schizophrenia include:
o Delusions of persecution (being persecuted against, e.g. ‘people are against me’).
o Delusions of reference (being referred to by others; e.g. ‘people are talking about me’).
o Delusions of grandeur (exaggerated self-importance; e.g. ‘I am God almighty’).
o Delusions of control (being controlled by an external force, known or unknown; e.g. ‘My
neighbour is controlling me”).
o Somatic (or hypochondriacal) delusions (e.g. ‘there are insects crawling in my scalp’).

Dermatology
102. What type of leprosy is associated with maximum antibody mediated immune response?
a. BB
b. BT
c. LL
d. TT

Ans: C, LL
Ref: Fitzpatrick’s Dermatology, 9th edition, page 2896

50
ONLINE PG NEPAL QAE Magh 1

 Leprosy clinical spectrum. Up to 80% of people exposed to M. leprae may solve the problem and get rid of
the bacilli before appearance of symptoms or after subclinical leprosy. Some patients will develop primary
neural leprosy, with no skin lesions. All those with skin lesions pass through an indeterminate form, and
then evolve to a polar tuberculoid leprosy (TT) or lepromatous leprosy (LL) disease or to an unstable
borderline form of leprosy. The paucibacillary (PB) pole toward TT has a good cellular immune response
(CIR), with the presence of Th1 cytokines, while multibacillary pole toward LL present an impaired CIR and
a high antibody response, with Th2 cytokines. Acid-fast bacilli and anti-PGL-I IgM are, both, low or
negative on PB and increase through multibacillary pole. Reversal reaction may happen especially in
borderline leprosy, whereas erythema nodosum leprosum occurs in borderline-lepromatous leprosy (BL)
and LL patients. Chronic neuritis or neuropathy may happen in primary neural leprosy and in all but the
indeterminate leprosy clinical form.

103. What is the whitish crescent-shaped base through the nail plate known as?
a. Cuticle
b. Lunula
c. Onychodermal band
d. Proximal nail fold

Ans: B, Lunula
Ref: Fitzpatrick’s Dermatology, 9th edition, page 107
 The nail plate is a hard structure that serves as a protective covering by preventing trauma to the tips of
toes and fingers; it is also used as a tool to pick up small objects, which is important for fine manipulations
and subtle finger functions. Through the nail plate, the whitish crescent-shaped base is called the lunula
(“small moon”), the visible part of the matrix. The lunula can best be seen in the thumb and may not be
visible in the little finger.

51
ONLINE PG NEPAL QAE Magh 1

104. Which of the following shows septal panniculitis?


a. Erythema induratum
b. Erythema nodosum
c. Lipodermatosclerosis
d. Lupus panniculitis

Ans: B, Erythema nodosum


Ref: Fitzpatrick’s Dermatology, 9th edition, page 1255

105. What is not correct about cutaneous response following exposure to UVB radiation?
a. Immediate pigment darkening is seen
b. Immunosuppression is produced
c. Involved in carcinogenesis
d. Involved in production of vitamin D

Ans: A, Immediate pigment darkening


Ref: Fitzpatrick’s Dermatology, 9th edition, page 269

52
ONLINE PG NEPAL QAE Magh 1

106. Which layer of epidermis is thickened in lichen planus?


a. Basal
b. Corneal
c. Granular
d. Pricky

Ans: C, Granular
Ref: Bolognia Dermatology, 4th edition, page 192
 The characteristic primary lesion of Lichen Planus is a small, polygonal-shaped, violaceous, flat-topped
papule; occasionally papules are umbilicated.
 The surface is slightly shiny or transparent, and a network of fine white lines called “Wickham striae” or
small gray–white puncta are also seen. The latter correspond histologically to focal thickening of the
granular layer. Wickham striae are readily apparent by dermoscopy.

107. What is the pH of normal skin?


a. 4.6 to 5.5
b. 5.6 to 6.4
c. 6.5 to 7.3
d. 7.3 to 7.5

Ans: B, 5.6 to 6.4


Ref: Fitzpatrick’s Dermatology, 9th edition, page 222
 Skin surface has antimicrobial properties.

53
ONLINE PG NEPAL QAE Magh 1

 Continuous epidermal cells turnover by desquamation prohibits colonization of microorganisms on the


skin.
 The physical properties of the skin surface itself prevents bacterial growth, that is, a low carbohydrate and
water content and a weakly acidic pH of 5.6 to 6.4.
 The weak acidic pH is due to various substances, such as free fatty acids secreted from sebaceous glands
or derived from phospholipid hydrolysis in the stratum corneum, lactic acid secreted from derived from
the eccrine glands, urocanic acid mostly derived from the degradation products of filaggrin, and
metabolites produced by microorganisms.

108. What is Auspitz sign a feature of?


a. Pemphigus
b. Photodermatitis
c. Pityriasis rosea
d. Psoriasis

Ans: D, Psoriasis
Ref: Fitzpatrick’s Dermatology, 9th edition, page 458
 The classic lesion of psoriasis is a well-demarcated, raised, red plaque with a white scaly surface. Lesions
can vary in size from pinpoint papules to plaques that cover large areas of the body.
 Under the scale, the skin has a glossy homogeneous erythema, and bleeding points appear when the scale
is removed, traumatizing the dilated capillaries below(the Auspitz sign).
 Psoriasis tends to be asymmetric eruption, and symmetry is a helpful feature in establishing a diagnosis.
Unilateral involvement can occur, however. The psoriatic phenotype may present a changing spectrum of
disease expression even within the same patient.

Radiology
109. What drug is used to enhance the image quality obtained in CT coronary angiography?
a. Beta blocker
b. Calcium channel blocker
c. Diuretic
d. N-acetyl cysteine

Ans: A, Beta blocker


Ref: Grainger & Allison’s Radiology, 6th edition, page 506
 Thanks to technological advances in the field of CCT and CMR, non-invasive coronary angiography has
become a reality and these novel techniques get integrated into daily clinical care. Current state-of-the-
artery multidetector CT (at least 64 or more slices) affords coronary artery imaging with sufficient spatial
and temporal resolution for clinical use. A typical clinical examination consists of an unenhanced CCT for
detection and quantification of coronary calcium, and a contrast enhanced CCT for coronary artery
imaging, detection of coronary artery plaques and, to some extent, characterization of the non-calcified
plaques.
 Patients are optimally suited if they have a regular heart rate and rhythm, a body mass index below 40
kg/m2 anda normal renal function. The examination is performed following intravenous injection of
contrast agent. Image quality can be substantially improved by lowering the patient’s heart rate to < 65

54
ONLINE PG NEPAL QAE Magh 1

bpm, which is usually achieved by administering β-blockers. Coronary vasodilatation can be achieved using
sublingual nitroglycerin administration.
 With the newest-generation imaging, not only can the heart be imaged in less than one heartbeat but also
the radiation can be substantially reduced(~0.7–3 mSv). Further reduction in radiation dose can be
achieved with iterative reconstruction algorithms.

110. What is not a common CT sign of pulmonary edema?


a. Ground glass opacification
b. Interlobular septal thickening
c. Mediastinal enlargement
d. Peribronchial vascular thickening

Ans: C, Mediastinal enlargement


Ref: Grainger & Allison’s Radiology, 6th edition, page 388
CT Signs of Pulmonary Oedema
 Common Findings
o Ground-glass opacification (patchy or diffuse) ± consolidation
o Smooth interlobular septal thickening
o Peri-bronchovascular thickening
o Vascular dilatation
 Ancillary Findings
o Pleural effusions
o Enlargement of mediastinal lymph nodes/'hazy' opacification of mediastinal fat (in heart
failure) reversible

111. What is the radiological imaging of choice for developmental dysplasia of hip in a newborn?
a. Bone scan
b. CT
c. MRI
d. USG

Ans: D, USG
Ref: Apley and Soloman’s Orthopaedics, 10 th edition, page 532
 Imaging in developmental dysplasia of hip: The introduction of ultrasound imaging, in the 1980s, allowed
visualization of the soft-tissue components of the infant hip including the cartilage of the femoral head
and acetabulum, the capsule and the labrum.
 The use of multiplanar and dynamic ultrasound enables visualization of the femoral head within the
acetabulum and assessment of the shape and depth of the acetabular cup.
 Ultrasound is best used for children before 6 months of age, after which ossification of structures makes
plain radiographs increasingly more helpful.

112. What is the sensitivity of specificity of CT in detecting pneumothorax respectively?


a. 50 % and 80%
b. 85% and 55%
c. 90% and 85%

55
ONLINE PG NEPAL QAE Magh 1

d. 100% and 100%

Ans: D, 100% and 100%


Ref: Grainger & Allison’s Radiology, 6th edition, page 368
 CT has a sensitivity and specificity for pneumothorax of almost 100%, with the only diagnostic difficulty
arising from differentiating pneumomediastinum from medial pneumothorax, when identification of the
mediastinal septation is key.

113. Magnetic resonance imaging depends upon which of the following atoms of the body to produce the images?
a. Calcium
b. Hydrogen
c. Oxygen
d. Sodium

Ans: B, Hydrogen
Ref: Learning Radiology - Recognizing the Basics, 3rd edition, page 5
 MRI utilizes the potential energy stored in the body’s hydrogen atoms.
 The atoms are manipulated by very strong magnetic fields and radiofrequency pulses to produce
enough localizing and tissue-specific energy to allow highly sophisticated computer programs to
generate two- and three- dimensional images.

114. What is the first line investigation for a suspected ovarian mass?
a. Contrast enhanced CT
b. HSG
c. MRI
d. Transvaginal USG

Ans: D, Transvaginal USG


Ref: Grainger & Allison’s Radiology, 6th edition, page 989
 Ninety per cent of all ovarian tumours are benign, although this varies with age. Transvaginal ultrasound is
the first-line investigation for a suspected ovarian mass.
 In most cases of non-complex masses, malignancy can be confidently excluded, but in 20% of cases the
ovarian mass remains indeterminate. The strength of MRI is its ability to further characterise adnexal
lesions.

115. Which of the following type of stone causing nephrolithiasis is radiolucent?


a. Calcium oxalate
b. Struvite
c. Xanthine
d. None

Ans: C, Xanthine
Ref: Grainger & Allison’s Radiology, 6th edition, page 833
 Nephrolithiasis refers to stone disease in the renal collecting system.

56
ONLINE PG NEPAL QAE Magh 1

 Calcium phosphate and calcium oxalate stones, which account for the majority of renal stones, are
radio-opaque, as are struvite stones, which represent 15% of renal stones, and develop in the setting
of infection and alkaline urine.
 Urate and xanthine stones represent 5% of renal stones and are radiolucent.

Anatomy
116. Which of the following embryological changes are seen in week 6 of development?
a. Auricular hillock and atrial septum formation
b. Digits and eyelids formation
c. External genitalia formation and fusion of facial prominences
d. Optic placode and lens placode formation

Ans: A, Auricular hillock and atrial septum formation


Ref: Langman’s Embryology, 14th edition, page v

117. What is oblique popliteal ligament an extension of?


a. Adductor magnus
b. Sartorius
c. Semimembranosus
d. Semitendinosus

Ans: C, Semimembranosus
Ref: Gray’s Anatomy, 41st edition, page 1989
 The knee joint capsule is a fibrous membrane of variable thickness. Anteriorly, it is replaced by the
patellar ligament and does not pass proximal to the patella or over the patellar area. Elsewhere, it lies
deep to expansions from vasti medialis and lateralis, separated from them by a plane of vascularized
loose connective tissue. The expansions are attached to the patellar margins and patellar ligament,
extending back to the corresponding collateral (tibial and fibular) ligaments and distally to the tibial
condyles. They form medial and lateral patellar retinacula, the lateral being reinforced by the iliotibial
tract.
 Posteriorly, the capsule contains vertical fibres that arise from the articular margins of the femoral
condyles and intercondylar fossa, and from the proximal tibia. The fibres mainly pass downwards and
somewhat medially. The oblique popliteal ligament is a well-defined thickening across the
posteromedial aspect of the capsule, and is one of the major extensions from the tendon of
semimembranosus.

118. The clavipectoral fascia thickens to form which of the following structure?
a. Coracoacromial
b. Coracoclavicular
c. Costoclavicular
d. Costocoracoid

Ans: D, Costocoracoid
Ref: Gray’s Anatomy, 41st edition, page 799

57
ONLINE PG NEPAL QAE Magh 1

 The clavipectoral fascia is the cranial continuation of the deep lamina of the pectoral fascia and the medial
continuation of the parietal layer of the subscapular bursal fascia. Laterally, it is continuous with the
coraco-acromial ligament above and lateral to the coracoid. It envelops the coracoid, short head of biceps
brachii and coracobrachialis. It covers the interval between those two muscles and pectoralis minor, which
it envelops, and then traverses the interval between pectoralis minor and subclavius. It splits around
subclavius and is attached to the clavicle anterior and posterior to the groove for subclavius. The posterior
layer is contiguous with the deep cervical fascia, a condensation of which forms a tether around the
central tendinous part of omohyoid, so indirectly connecting it to the clavicle. The deep aspect of the
posterior lamina of the clavipectoral fascia blends with the sheath of the axillary vessels. Medially, it
blends with the fascia over the first two intercostal spaces and is attached to the first rib, medial to
subclavius.
 Occasionally, the fascia thickens to form a band between the first rib and coracoid process, the
costocoracoid ligament, under which the lateral cord of the brachial plexus is closely applied. The cephalic
vein, thoraco-acromial artery and associated veins and lymphatic vessels, and the lateral pectoral nerve all
pass through the fascia, immediately cranial to the upper border of pectoralis minor.

119. Which of the following muscle doesn’t belong to the scapulohumeral functional muscle group of the shoulder?
a. Infraspinatus
b. Subscapularis
c. Teres minor
d. Trapezius

Ans: D, Trapezius
Ref: Gray’s Anatomy, 41st edition, page 816

120. What type of joint is the carpometacarpal joint of the thumb?

58
ONLINE PG NEPAL QAE Magh 1

a. Ellipsoid joint
b. Hinge joint
c. Planar joint
d. Saddle joint

Ans: D, Saddle joint


Ref: Gray’s Anatomy, 41st edition, page 876
The carpometacarpal joint of the thumb is a sellar (saddle) joint between the first metacarpal base and trapezium. Its
extensive articular surfaces and their topology ensure that the joint enjoys a wide mobility.

121. Lesser trochanter of femur forms the insertion site for which of the following muscles?
a. Iliacus
b. Rectus femoris
c. Sartorius
d. Vastus medialis

Ans: A, Iliacus
Ref: Gray’s Anatomy for Students, 4th edition, page 582

122. What is represented by the acute margin of the heart?


a. Inferior border of the heart
b. Left border of the heart
c. Right border of the heart
d. Upper border of the heart

Ans: A, Inferior border of the heart


Ref: Gray’s Anatomy, 41st edition, page 1000

59
ONLINE PG NEPAL QAE Magh 1

 Upper border of the heart: This is atrial (mainly the left atrium). Anterior to it are the ascending aorta and
the pulmonary trunk. At its extremity, the superior vena cava enters the right atrium.
 Right border of the heart: Corresponding to the right atrium, the profile of the right border is slightly
convex to the right as it approaches the vertical.
 Inferior border of the heart: Also known as the acute margin of the heart, the inferior border is sharp, thin
and nearly horizontal. It extends from the lower limit of the right border to the apex and is formed mainly
by the right ventricle, with a small contribution from the left ventricle near the apex.
 Left border of the heart: Also known as the obtuse margin, the left border separates the sternocostal and
left surfaces. It is round and mainly formed by the left ventricle but, to a slight extent superiorly, is formed
by the left atrial appendage. It descends obliquely, convex to the left, from the appendage to the cardiac
apex.

123. Which among the following does not drain into the azygos vein?
a. Accessory hemiazygos vein
b. Esophageal vein
c. Left bronchial vein
d. Mediastinal vein

Ans: C, Left bronchial vein


Ref: Gray’s Anatomy for Student’s, 4th edition, page 231
 Tributaries of the azygos vein include:
o the right superior intercostal vein (a single vessel formed by the junction of the second, third,
and fourth intercostal veins),
o fifth to eleventh right posterior intercostal veins,
o the hemiazygos vein,
o the accessory hemiazygos vein,
o esophageal veins,
o mediastinal veins,
o pericardial veins, and
o right bronchial veins.

124. What is the ligament of Mackenrodt’s?


a. Broad ligament
b. Round ligament
c. Transverse cervical ligament
d. Uterosacral ligament

Ans: C, Transverse cervical ligament


Ref: Gray’s Anatomy, 41st edition, page 1298
 The uterosacral, transverse cervical and pubocervical ligaments are condensations of the visceral or
endopelvic connective tissue that connect the pelvic viscera to the side wall of the pelvis; they radiate like
the spokes of a wheel around the hub of the cervix, providing it with considerable support. The connective
tissue lateral to the uterus and the cervix – the parametrium – continues down along the vagina as the
paracolpium.

60
ONLINE PG NEPAL QAE Magh 1

 The uterosacral ligaments contain fibrous tissue and smooth muscle. They pass back from the cervix and
uterine body on both sides of the rectum, and are attached to the anterior aspect of the sacrum. They can
be palpated laterally on rectal examination and can be felt as thick bands of tissue passing downwards on
both sides of the posterior fornix on vaginal examination.
 The transverse cervical ligaments (cardinal ligaments, ligaments of Mackenrodt) extend from the side of
the cervix and lateral fornix of the vagina, and are attached extensively on the pelvic wall.
 The lower parts of the ureters and pelvic blood vessels traverse the transverse cervical ligaments. Fibres of
the pubocervical ligament pass forwards from the anterior aspect of the cervix and upper vagina to
diverge around the urethra, and are attached to the posterior aspect of the pubic bones.
 The transverse cervical and uterosacral ligaments are almost vertically orientated in the standing position
and maintain the near-horizontal axis of the upper vagina. The uterus and vagina are supported by the
close interaction of the uterosacral and transverse cervical ligaments with the muscles of the pelvic floor,
including the levatores ani and coccygei, the perineal membrane and the perineal body.

125. What is false about hepatic innervation?


a. Dual innervation
b. Hepatic plexus supplies the parenchyma
c. Referred pain occurs at right shoulder region
d. Upper intercostal nerves supply the capsule

Ans: D, Upper intercostal nerves supply the capsule


Ref: Gray’s Anatomy, 41st edition, page 1169
 The liver has a dual innervation. The parenchyma is supplied by nerves arising from the hepatic plexus,
which contains sympathetic and parasympathetic (vagal) fibres; they all enter the liver at the porta
hepatis. The capsule is supplied by fine branches of the lower intercostal nerves, which also supply the
parietal peritoneum, particularly around the ‘bare area’ and superior surface; distension or disruption of
the liver capsule causes quite well-localized, sharp pain.
 The hepatic plexus receives preganglionic parasympathetic fibres from the anterior and, to a lesser extent,
the posterior vagus, and postganglionic sympathetic fibres via the coeliac and superior mesenteric
plexuses.
 Pain arising from the parenchyma of the liver is poorly localized. In common with other structures of
foregut origin, pain is referred to the epigastrium. Stretch or irritation of the liver capsule by inflammation
or neoplasia produces well-localized ‘somatic’ pain. Pathology involving the diaphragmatic surface of the
liver may be referred via the phrenic nerve to the right shoulder region (C3,4,5 dermatomes).

126. What is incorrect about substantia nigra?


a. Extends through the entire length of the midbrain
b. It is located between the crus cerebri ventrolaterally and ascending lemniscal fibres dorsomedially
c. Medial part is traversed by oculomotor axons
d. Pars compacta is located in the ventral aspect

Ans: D, Pars compacta is located in the ventral aspect


Ref: Gray’s Anatomy, 41st edition, page 322 – 323
 The substantia nigra is a large nucleus that extends through the entire length of the midbrain, located
between the crus cerebri ventrolaterally and ascending lemniscal fibres dorsomedially.

61
ONLINE PG NEPAL QAE Magh 1

 Its medial part is traversed by oculomotor axons passing to their point of exit in the interpeduncular fossa.
 The substantia nigra is divided into a larger part consisting of a dorsal pars compacta and a ventral pars
reticularis and a smaller pars lateralis.

127. Which of the following is the medial vestibular nuclei?


a. Bechterew
b. Deiters
c. Reiters
d. Schwalbe

Ans: D, Schwalbe
Ref: Gray’s Anatomy, 41st edition, page 319 – 320
 The vestibular complex consists of the medial (Schwalbe), lateral (Deiters’ nucleus), superior (Bechterew)
and inferior vestibular nuclei.
 The medial vestibular nucleus is the largest. It broadens, and then narrows as it ascends from the upper
olivary level into the lower pons, where it is located lateral and superior to visceromotor nuclei of cranial
nerves IX and X. It is crossed by the striae medullares, running nearer the floor, and is continuous with the
nucleus intercalatus below.

Physiology
128. Which ion channel is closed in the sequence of events involved in phototransduction in rods and cones?
a. Calcium
b. Hydrogen
c. Potassium
d. Sodium

Ans: D, Sodium
Ref: Ganong’s Review of Physiology, 25th edition, page 184

62
ONLINE PG NEPAL QAE Magh 1

129. What is the reaction time for knee jerk reflex in humans?
a. 6 – 12 ms
b. 12 – 18 ms
c. 18 – 24 ms
d. 24 – 30 ms

Ans: C, 18 – 24 ms
Ref: Ganong’s Review of Physiology, 25th edition, page 230
The time between the application of the stimulus and the response is called the reaction time. In humans, the reaction
time for a stretch reflex such as the knee jerk is 19–24 ms.

130. Gravity is detected by which of the following special sense receptors?


a. Cochlea
b. Otolith organs
c. Semicircular canals
d. Vestibular organs

Ans: B, Otolith organs


Ref: Ganong’s Review of Physiology, 25th edition, page 160

63
ONLINE PG NEPAL QAE Magh 1

131. In which layer of the cerebellum are Basket cells present?


a. Granular layer
b. Purkinje layer
c. Molecular layer
d. All of the above

Ans: C, Molecular layer


Ref: Ganong’s Review of Physiology, 25th edition, page 249
 Basket cells (Figure 12–17) are located in the molecular layer. They receive excitatory input from the
parallel fibers and each projects to many Purkinje cells. Their axons form a basket around the cell body
and axon hillock of each Purkinje cell they innervate.

132. What is present in the medial most aspect of the sensory homunculus along the post central gyrus?
a. Face
b. Genitalia
c. Hand
d. Trunk

Ans: B, Genitalia
Ref: Ganong’s Review of Physiology, 25th edition, page 170

64
ONLINE PG NEPAL QAE Magh 1

133. What is not a hallmark of Horner syndrome?


a. Dilated pupils
b. Drooping eyelids
c. Reduced sweating
d. Sunken eyeball

Ans: A, Dilated pupils


Ref: Ganong’s Review of Physiology, 25th edition, page 263
 Horner syndrome is a rare disorder resulting from interruption of preganglionic or postganglionic
sympathetic innervation to the face. The problem can result from injury to the nerves, injury to the carotid
artery, a stroke or lesion in the brainstem, or a tumor in the lung. In most cases the problem is unilateral,
with symptoms occurring only on the side of the damage.
 The hallmark of Horner syndrome is the triad of anhidrosis (reduced sweating), ptosis (drooping eyelid),
and miosis (constricted pupil). Symptoms also include enophthalmos (sunken eyeball) and vasodilation.

134. What is the equilibrium potential of bicarbonate ion?


a. +65 mv
b. -12 mv
c. -32 mv

65
ONLINE PG NEPAL QAE Magh 1

d. -62 mv

Ans: C, -32 mv
Ref: Ganong’s Review of Physiology, 25th edition, page 103

135. What is false about the skeletal muscle protein titin?


a. It is the largest known protein
b. Connects the Z lines to the I lines
c. Provides muscle with its elasticity
d. Scaffolds the sarcomere

Ans: B, Connects the Z lines to the I lines


Ref: Ganong’s Review of Physiology, 25th edition, page 101
 Titin, the largest known protein (with a molecular mass near 3,000,000 Da), connects the Z lines to the M
lines and provides scaffolding for the sarcomere. It contains two kinds of folded domains that provide
muscle with its elasticity. At first when the muscle is stretched there is relatively little resistance as the
domains unfold, but with further stretch there is a rapid increase in resistance that protects the structure
of the sarcomere.

136. What is not produced by the stimulation of κ opioid receptor?


a. Diuresis
b. Dysphoria
c. Respiratory depression
d. Spinal analgesia

Ans: C, Respiratory depression


Ref: Ganong’s Review of Physiology, 25th edition, page 154

66
ONLINE PG NEPAL QAE Magh 1

137. What is the maximum distance present in the human body between a tissue and a capillary?
a. 25 micrometers
b. 50 micrometers
c. 100 micrometers
d. 150 micrometers

Ans: B, 50 micrometers
Ref: Guyton and Hall Physiology, 13th edition, page 457
 Chemotaxis depends on the concentration gradient of the chemotactic substance. The concentration is
greatest near the source, which directs the unidirectional movement of the WBCs.
 Chemotaxis is effective up to 100 micrometers away from an inflamed tissue. Therefore, because almost
no tissue area is more than 50 micrometers away from a capillary, the chemotactic signal can easily move
hordes of WBCs from the capillaries into the inflamed area.

138. What is not an inhibitory input to rostral ventrolateral medulla?


a. Brainstem reticular formation
b. Cardiopulmonary baroreceptors
c. Caudal medullary raphe nuclei
d. Lung inflation afferents

Ans: A, Brainstem reticular formation


Ref: Ganong’s Review of Physiology, 25th edition, page 589

67
ONLINE PG NEPAL QAE Magh 1

139. What is the ratio of precursor red blood cells to precursor white blood cells in bone marrow?
a. 25% : 75%
b. 50% : 50%
c. 75% : 25%
d. 90% : 10%

Ans: A, 25% : 75%


Ref: Ganong’s Review of Physiology, 25th edition, page 554
 Normally, 75% of the cells in the marrow belong to the white blood cell–producing myeloid series and only
25% are maturing red cells, even though there are over 500 times as many red cells in the circulation as
there are white cells. This difference in the marrow reflects the fact that the average life span of white
cells is short, whereas that of red cells is long.

Pathology
140. What is not a feature of irreversible hypoxic-ischemic neuronal injury?
a. Absence of Nissl substance
b. Intense basophilia of the cytoplasm
c. Loss of nucleolus
d. Pyknosis of the nucleus

Ans: B, Intense basophilia of the cytoplasm


Ref: Robbins Basic Pathology, 10th edition, page 849
 Features of Neuronal Injury. In response to injury, a number of changes occur in neurons and their
processes (axons and dendrites). Within 12 hours of an irreversible hypoxic-ischemic insult, neuronal
injury becomes evident on routine hematoxylin and eosin (H&E) staining.
 There is shrinkage of the cell body, pyknosis of the nucleus, disappearance of the nucleolus, loss of Nissl
substance, and intense eosinophilia of the cytoplasm (“red neurons”). Axonal injury also leads to cell body

68
ONLINE PG NEPAL QAE Magh 1

enlargement and rounding, peripheral displacement of the nucleus, enlargement of the nucleolus, and
peripheral dispersion of Nissl substance (central chromatolysis).
 In addition, acute injuries typically result in breakdown of the blood-brain barrier and variable degrees of
cerebral edema.

141. Recurrent HSV-1 encephalitis is mostly associated with inherited mutation of signaling of which type of toll like
receptor?
a. TLR 1
b. TLR 2
c. TLR 3
d. TLR 4

Ans: C, TLR 3
Ref: Robbins Basic Pathology, 10th edition, page 866
Recurrent HSV-1 encephalitis is sometimes associated with inherited mutations that interfere with Toll-like receptor
signaling (specifically that of TLR-3), which has an important role in anti-viral defense.

142. What is the commonest cause of right sided heart failure?


a. Left sided heart failure
b. Obstructive sleep apnea
c. Pulmonary hypertension
d. Pulmonary thromboembolism

Ans: A, Left sided heart failure


Ref: Robbins Basic Pathology, 10th edition, page 402
 Right-sided heart failure is usually the consequence of left-sided heart failure, since any pressure increase
in the pulmonary circulation inevitably produces an increased burden on the right side of the heart.
Consequently, the causes of right-sided heart failure include all of those that induce left-sided heart
failure.
 Isolated right-sided heart failure is infrequent and typically occurs in patients with one of a variety of
disorders affecting the lungs; hence it is often referred to as cor pulmonale.

143. What is the most common cause of rhythm disorders of the heart?
a. Congenital abnormalities
b. Drugs
c. Ischemia
d. Myocarditis

Ans: C, Ischemia
Ref: Robbins Basic Pathology, 10th edition, page 420
Ischemic injury is the most common cause of rhythm disorders, either through direct damage or through the dilation of
heart chambers that alters signal conduction.

144. What is coronary artery spasm associated with?


a. Crescendo angina

69
ONLINE PG NEPAL QAE Magh 1

b. Stable angina
c. Unstable angina
d. Variant angina

Ans: D, Variant angina


Ref: Robbins Basic Pathology, 10th edition, page 411
 Angina pectoris is an intermittent chest pain caused by transient, reversible myocardial ischemia. The pain
is a consequence of the ischemia-induced release of adenosine, bradykinin, and other molecules that
stimulate autonomic nerves. The following three variants are recognized:
 Typical or stable angina is predictable episodic chest pain associated with particular levels of exertion or
some other increased demand (e.g., tachycardia). The pain is described as a crushing or squeezing
substernal sensation that often radiates down the left arm or to the left jaw (referred pain). The pain
usually is relieved by rest (reducing demand) or by drugs such as nitroglycerin, a vasodilator that increases
coronary perfusion.
 Prinzmetal or variant angina occurs at rest and is caused by coronary artery spasm. Although such spasms
typically occur on or near existing atherosclerotic plaques, a completely normal vessel can be affected.
Prinzmetal angina typically responds promptly to vasodilators such as nitroglycerin and calcium channel
blockers.
 Unstable angina (also called crescendo angina) is characterized by increasingly frequent pain, precipitated
by progressively less exertion or even occurring at rest. Unstable angina is associated with plaque
disruption and superimposed thrombosis, distal embolization of the thrombus, and/or vasospasm; it can
be a harbinger of MI, portending complete vascular occlusion.

145. What is the leading cause of atherosclerosis?


a. Cigarette smoking
b. Diabetes
c. Hypercholesterolemia
d. Hypertension

Ans: C, Hypercholesterolemia
Ref: Robbins Basic Pathology, 10th edition, page 371
 Hyperlipidemia—and, more specifically, hypercholesterolemia—is a major risk factor for development of
atherosclerosis and is sufficient to induce lesions in the absence of other risk factors.
 The main cholesterol component associated with increased risk is low-density lipoprotein (LDL)
cholesterol (“bad cholesterol”); LDL distributes cholesterol to peripheral tissues. By contrast, high-density
lipoprotein (HDL) cholesterol (“good cholesterol”) mobilizes cholesterol from developing and existing
vascular plaques and transports it to the liver for biliary excretion. Consequently, higher levels of HDL
correlate with reduced risk. Recognition of these relationships has spurred the development of dietary
and pharmacologic interventions that lower total serum cholesterol or LDL and/or raise serum HDL

146. What is the leading cause of primary brain parenchymal hemorrhage?


a. Hypertension
b. Ruptured aneurysm
c. Trauma
d. Tumors

70
ONLINE PG NEPAL QAE Magh 1

Ans: A, Hypertension
Ref: Robbins Basic Pathology, 10th edition, page 854
 Primary Brain Parenchymal Hemorrhage: Spontaneous (nontraumatic) intraparenchymal hemorrhages are
most common in mid to late adult life, with a peak incidence at about 60 years of age. Most are due to the
rupture of a small intraparenchymal vessel.
 Hypertension is the leading underlying cause, and brain hemorrhage accounts for roughly 15% of deaths
among individuals with chronic hypertension.

147. Which lung tumor is associated with collar-button lesion?


a. Adenocarcinoma
b. Carcinoid
c. Small cell carcinoma
d. Squamous cell carcinoma

Ans: B, Carcinoid
Ref: Robbins Basic Pathology, 10th edition, page 543
 Most carcinoids originate in main bronchi and grow in one of two patterns: (1) an obstructing polypoid,
spherical, intraluminal mass, or (2) a mucosal plaque penetrating the bronchial wall to fan out in the
peribronchial tissue—the so-called collar-button lesion.
 Even penetrating lesions push into the lung substance along a broad front and are well demarcated.
Peripheral carcinoids are less common. Although 5% to 15% of carcinoids have metastasized to the hilar
nodes at presentation, distant metastases are rare. Histologically, typical carcinoids, like their
counterparts in the intestinal tract, are composed of nests of uniform cells that have regular round nuclei
with “salt-and-pepper” chromatin, absent or rare mitoses and little pleomorphism.
 Atypical carcinoid tumors display a higher mitotic rate and small foci of necrosis. These tumors have a
higher incidence of lymph node and distant metastasis than typical carcinoids. Unlike typical carcinoids,
the atypical tumors have TP53 mutations in 20% to 40% of cases.
 Typical carcinoid, atypical carcinoid, and large cell neuroendocrine and small cell carcinoma can be viewed
as a continuum of increasing histologic aggressiveness and malignant potential within the spectrum of
pulmonary neuroendocrine neoplasms.

148. Which type of emphysema is seen in α1-anti-trypsin deficiency?


a. Centrilobular emphysema
b. Distal lobular emphysema
c. Panlobular emphysema
d. Paraseptal emphysema

Ans: C, Panlobular emphysema


Ref: Robbins Basic Pathology, 10th edition, page 499
 Centriacinar (centrilobular) emphysema: The central or proximal parts of the acini, formed by respiratory
bronchioles, are affected, while distal alveoli are spared. This type of emphysema is most common in cigarette
smokers, often in association with chronic bronchitis.
 Panacinar (panlobular) emphysema: The acini are uniformly enlarged, from the level of the respiratory
bronchiole to the terminal blind alveoli. It is associated with α1-anti-trypsin deficiency.

71
ONLINE PG NEPAL QAE Magh 1

 Distal acinar (paraseptal) emphysema: The proximal portion of the acinus is normal but the distal part is
primarily involved. The cause of this type of emphysema is unknown; it is common in young adults who
present with spontaneous pneumothorax

149. A small posterior fossa with a misshapen midline cerebellum and downward extension of the vermis through
the foramen magnum, hydrocephalus and a lumbar myelomeningocele is seen in which of the following
congenital malformation?
a. Arnold-Chiari malformation
b. Chiari type I malformation
c. Chiari type III malformation
d. Dandy-Walker malformation

Ans: A, Arnold-Chiari malformation


Ref: Robbins Basic Pathology, 10th edition, page 861
 Arnold-Chiari malformation (Chiari type II malformation) combines a small posterior fossa with a
misshapen midline cerebellum and downward extension of the vermis through the foramen magnum;
hydrocephalus and a lumbar myelomeningocele typically are also present.
 The far milder Chiari type I malformation has low-lying cerebellar tonsils that extend through the foramen
magnum. Excess tissue in the foramen magnum results in partial obstruction of CSF flow and compression
of the medulla, with symptoms of headache or cranial nerve deficits often manifesting only in adult life.
Surgical intervention can alleviate the symptoms.
 Dandy-Walker malformation is characterized by an enlarged posterior fossa, absence of the cerebellar
vermis, and a large midline cyst.

Pharmacology
150. What is not a type A drug reaction?
a. Allergy
b. Drug withdrawal effect
c. Side effect
d. Toxic effect

Ans: A, Allergy
Ref: KDT Pharmacology, 8th edition, page 92
 Predictable (Type A or Augmented) reactions (mechanism based adverse reactions): These are based on
the pharmacological properties of the drug, which means that they are augmented, but qualitatively
normal response to the drug; include side effects, toxic effects and consequences of drug withdrawal.
They are more common, dose related and mostly preventable and reversible.
 Unpredictable (Type B or Bizarre) reactions These are based on peculiarities of the patient and not on
drug's known actions; include allergy and idiosyncrasy. They are less common, often non-dose related,
generally more serious and require withdrawal of the drug. Some of these reactions can be predicted and
prevented if their genetic basis is known and suitable test to characterize the individual’s phenotype is
performed.

151. What percentage of adult dose of a drug is usually needed in a 7 year old child?
a. 10%

72
ONLINE PG NEPAL QAE Magh 1

b. 30%
c. 50%
d. 75%

Ans: C, 50%
Ref: KDT Pharmacology, 8th edition, page 74

152. Administration of drug via which route requires the lowest dose of drug?
a. Dermajet
b. Intramuscular
c. Intravenous
d. Nebulization

Ans: C, Intravenous
Ref: KDT Pharmacology, 8th edition, page 14
 The drug is injected asa bolus (Greek: bolos- lump) or infused slowly over hours in one of the superficial
veins. The drug reaches directly into the blood stream and effects are produced immediately (great value
in emergency).
 The dose of the drug required s smallest (bioavailability is 100%) and evenlarge volumes can be infused.

153. What is not a central action of morphine?


a. Analgesia
b. Hypotension
c. Inhibition of Edinger-Westphal nucleus
d. Stimulation of medullary vagal center

Ans: C, Inhibition of Edinger-Westphal nucleus


Ref: KDT Pharmacology, 8th edition, page 499
 The stimulant central actions of morphine are:
o Sensitization of CTZ to vestibular and other impulses, causing nausea and vomiting.
o Stimulation of Edinger-Westphal nucleus of the 3rd cranial nerve by inhibiting the GABAergic
interneurone which keeps this nucleus tonically inhibited, producing miosis. Note that miosis occurs
on topical application of morphine to the eye.

73
ONLINE PG NEPAL QAE Magh 1

o Stimulates the medullary vagal center producing bradycardia as the usual response.
o Certain cortical areas and hippocampal cells are stimulated causing truncal rigidity and immobility at
high doses.
o Other central actions of morphine are:
 Analgesia
 Sedation
 Subjective effects and euphoria
 Respiratory depression
 Cough suppression
 Temperature regulation - Hypothermia
 Vasomotor regulation - Hypotension

154. Which is not a purine antagonist?


a. 6-mercaptopurine
b. 6-thioguanine
c. Fludarabine
d. Gemcitabine

Ans: D, Gemcitabine
Ref: KDT Pharmacology, 8th edition, page 916-917

155. A patient with hypertension has been managed with diuretics. What is the most common adverse effect to
develop in him?
a. Hyperglycemia
b. Hyperlipidemia
c. Hyponatremia
d. Hypokalemia

74
ONLINE PG NEPAL QAE Magh 1

Ans: D, Hypokalemia
Ref: Katzung Pharmacology, 14th edition, page 178
 In treatment of hypertension, the most common adverse effect of diuretics (except potassium-sparing diuretics) is
potassium depletion. Although mild degrees of hypokalemia are tolerated well by many patients, hypokalemia
may be hazardous in persons taking digitalis, those who have chronic arrthymias, or those with acute myocardial
infarction of left ventricular dysfunction.
 Potassium loss is coupled to reabsorption of sodium, and restriction of dietary sodium intake therefore minimizes
potassium loss.
 Diuretics may also cause magnesium depletion, impair glucose tolerance and increase serum lipid concentrations.
 Diuretics increase uric acid concentrations and may precipitate gout.

156. Which among the following histamine receptor acts via inositol triphosphate and diacylglycerol?
a. H1
b. H2
c. H3
d. H4

And: A, H1
Ref: Katzung Pharmacology, 14th edition, page 279

157. Which is a class1B antiarrhythmic agent?


a. Amiodarone
b. Diltiazem
c. Mexiletine
d. Procainamide

Ans: C, Mexileine
Ref: Katzung Pharmacology, 14th edition, page 237
 Antiarrhythmic agents are recognized as 4 classes:
 Class 1 action is sodium and calcium channel blockade. IT is subdivided on basis of effect on action
potential duration (APD).
o Class1A (quinidine, procainamide) prolongs APD.
o Class1B (lidocaine, mexiletine) shortens APD.
o Class1C (propafenone, flecainide) have minimal effect on APD.

75
ONLINE PG NEPAL QAE Magh 1

 Class 2 (propranolol, esmolol) action is sympatholytic


 Class 3 (amiodarone, dofetilide) drugs block the rapid component of the delayed rectifier potassium
current. It prolongs APD.
 Class 4 (diltiazem, verapamil) blocks cardiac calcium current.

158. Which is a halogen antiseptic?


a. Chlorhexidine
b. Chlorophores
c. Chloroxylenol
d. Cresol

Ans: B, Chlorophore
Ref: KDT Pharmacology, 8th edition, page 958
 The classification of antiseptics is:
o Phenol derivative: phenol, cresol, chloroxylenol, hexachlorophene
o Oxidizing agent: potassium permanganate, hydrogen peroxide, benzoyl peroxide
o Halogens: iodine, iodophores, chlorine, chlorophores
o Biguanides: chlorhexidine
o Quaternary ammonium (cationic): cetrimide, benzalkonium chloride, dequalinium chloride
o Soaps: of sodium and potsaaium
o Alcholols: ethanol, isopropranol
o Aldehydes: formaldehyde, glutaraldehyde
o Acids: boric acid

159. A patient on anti-tubercular therapy develops neurotoxicity. How will you manage him?
a. Pyridoxine 10 mg/day
b. Pyridoxine 100 mg/day
c. Riboflavin 10 mg/day
d. Riboflavin 100 mg/day

Ans: B, Pyridoxine 100 mg/day


Ref: KDT Pharmacology, 8th edition, page 818
 Peripheral neuritis and a variety of neurological manifestations (paresthesias, numbness, mental disturbances,
and rarely convulsions) are the most important dose-dependent toxic effects. These are due to interference
with production of the active coenzyme pyridoxal phosphate from pyridoxine, and its increased excretion in
urine. Pyridoxine given prophylactically in dose of 10mg/day prevents the neurotoxicity even with higher
doses of INH.
 INH neurotoxicity is treated with pyridoxine 100 mg/day.

76
ONLINE PG NEPAL QAE Magh 1

Biochemistry
160. What is a nutritionally semi-essential amino acid?
a. Arginine
b. Asparagine
c. Isoleucine
d. Leucine

Ans: A, Arginine
Ref: Harper’s Biochemistry, 30th edition, page 282

161. What type of body protein is mostly turned over by humans every day?
a. Blood cells
b. Epithelial proteins
c. Muscle proteins
d. Neural proteins

Ans: C, Muscle proteins


Ref: Harper’s Biochemistry, 30th edition, page 287 – 288
 The continuous degradation and synthesis (turnover) of cellular proteins occur in all forms of life.
 Each day, humans turnover 1% to 2% of their total body protein, principally muscle protein.
 High rates of protein degradation occur in tissues that are undergoing structural rearrangement, for
example, uterine tissue during pregnancy, skeletal muscle in starvation, and tadpole tail tissue during
metamorphosis.
 While approximately75% of the amino acids liberated by protein degradation are reutilized, the
remaining excess free amino acids are not stored for future use. Amino acids not immediately
incorporated into new protein are rapidly degraded. The major portion of the carbon skeletons of the
amino acids is converted to amphibolic intermediates, while in humans the amino nitrogen is
converted to urea and excreted in the urine.

162. Pantothenate is present in which coenzyme?

77
ONLINE PG NEPAL QAE Magh 1

a. Amino acidadenylates
b. Coenzyme A
c. FAD
d. NADP

Ans: B, Coenzyme A
Ref: Harper’s Biochemistry, 30th edition, page 344

163. What is false about von Gierke Disease?


a. Glucose-6-phosphatase deficiency
b. Hypouricemia
c. Increased ribose 5-phosphate
d. Lactic acidosis

Ans: B, Hypouricemia
Ref: Harper’s Biochemistry, 30th edition, page 355
 von Gierke Disease: Purine overproduction and hyperuricemia in von Gierke disease(glucose-6-
phosphatase deficiency) occurs secondary to enhanced generation of the PRPP precursor ribose 5-
phosphate. An associated lactic acidosis elevates the renal threshold for urate, elevating total body urates.

164. Bile salts are detected by which test?


a. Rothera’s test

78
ONLINE PG NEPAL QAE Magh 1

b. Gmelin’s test
c. Fouchet’s test
d. Pettenkofer’s test

Ans: D, Pettenkofer’s test


Ref: Satyanarayana Biochemistry, 4th edition, page 766
 Rothera’s test: Nitroprusside in alkaline medium reacts with keto group of ketone bodies (acetone and
acetoacetate) to form a purple ring. The test is not given by β-hydroxybutyrate.
 Gmelin’s test: Nitric acid oxidizes the bile pigment bilirubin to biliverdin (green) or bilicyanin (blue). Gmelin’s
test gives a play of colors and is used for the identification of bile pigments.
 Fouchet’s test: This test is also employed for the detection of bile pigments. Bile pigments are adsorbed on
barium sulfate. Fouchet’s reagents (containing ferric chloride in trichloroacetic acid) oxidizes bilirubin to
biliverdin and bilicyanin.
 Pettenkofer’s test: This test is employed for the detection of bile salts. The furfural derivatives (by reacting
sugar with concentrated sulfuric acid) condenses with bile salts to form a purple ring.

165. What is the function of siRNAs?


a. Apoptosis induction
b. Inhibition of gene expression
c. Synthesis of proteins
d. Transfer amino acids

Ans: B, Inhibition of gene expression


Ref: Harper’s Biochemistry, 30th edition, page 368
The small ncRNAs termed miRNAs and siRNAs typically inhibit of gene expression at the level of specific protein production
by targeting mRNAs through one of several distinct mechanisms.

166. What is deficient in Keshan disease?


a. Cysteine
b. Iron
c. Selenium
d. Magnesium

Ans: C, Selenium
Ref: Harper’s Biochemistry, 30th edition, page 286
Impairments in human selenoproteins have been implicated in tumorigenesis and atherosclerosis, and are associated with
selenium deficiency cardiomyopathy(Keshan disease).

167. What is standard free energy of hydrolysis of ATP?


a. −9.2 KJ/mol
b. −30.5 KJ/mol
c. −43.2 KJ/mol
d. −61.9 KJ/mol

Ans: B, −30.5 KJ/mol

79
ONLINE PG NEPAL QAE Magh 1

Ref: Harper’s Biochemistry, 30th edition, page 116

Microbiology
168. What is Thayer-Martin medium used to isolate?
a. Hemophilus influenzae
b. Neisseria gonorrheae
c. Proteus mirabilis
d. Streptococcus pyogenesis

Ans: B, Neisseria gonorrheae


Ref: Ananthanarayan Microbiology, 10th edition, page 42
 The Thayer-Martin medium containing antimicrobials (vancomycin 3.0 mg, colistin 7.5 mg and
nystatin12.5 units per ml of agar) in chocolate agar is usedto isolate N. gonorrhoeae from clinical
specimens.
 The antimicrobials suppress the growth of othercommensal organisms which may inhibit the growth
ofN.gonorrhoeae. Thayer-Martin plates are incubated inan atmosphere containing 3 – 10% CO 2

169. Anaerobes are most widely cultured in which medium?


a. BACTEC
b. McIntosh and Filde's anaerobic jar
c. Robertson' s cooked meat medium
d. Thioglycollate broth

Ans: C, Robertson' s cooked meat medium


Ref: Ananthanarayan Microbiology, 10th edition, page 46

80
ONLINE PG NEPAL QAE Magh 1

 Robertson' s cooked meat medium is probably the most widely used fluid medium for the culture of
anaerobes. It consists of fat-free minced cooked meat in broth.
 It permits the growth of even strict anaerobes and indicates their saccharolytic and proteolytic activities
by the meat being turned red or black, respectively.

170. What is the smallest unit of antigenicity?


a. Cotope
b. Epitope
c. Paratope
d. Prostotope

Ans: B, Epitope
Ref: Ananthanarayan Microbiology, 10th edition, page 90
The smallest unit of antigenicity is known as the antigenic determinant or epitope.

171. What phase of bacterial growth shows Gram variability?


a. Lag
b. Log
c. Stationary
d. Decline

Ans: C, Stationary
Ref: Ananthanarayan Microbiology, 10th edition, page 23
 The various stages of the growth curve are associated with morphological and physiological
alterations of the cells.
o Lag phase maximum cell size is obtained towards the end of the lag phase.
o Log phase, cells are smaller and stain uniformly.
o Stationary phase, cells are frequently Gram variable and show irregular staining due to the
presence of intracellular storage granules. Sporulation occurs at this stage. Also, many
bacteria produce secondary metabolic products such as exotoxins and antibiotics.
o Phase of decline involution forms are common

172. Resistance to which of the following agent correlates with the presence of mec A gene in Staphylococci?
a. Cefoxitin
b. Ceftriaxone
c. Methicillin
d. Vancomycin

Ans: A, Cefoxitin
Ref: Ananthanarayan Microbiology, 10th edition, page 207
 Detection of MRSA: This is done by cefoxitin disc diffusion agar as per the present international
guidelines. Cefoxitin resistance correlates with the presence of mec A gene.
 Some strains called borderline resistance S. aureus(BORSA) have been found to be missed by cefoxitin
susceptibility test and may need to be tested under special incubation conditions using oxacillin
screen agar.

81
ONLINE PG NEPAL QAE Magh 1

173. What is Pneumocystis jiroveci?


a. Bacteria
b. Fungi
c. Protozoa
d. Virus

Ans: B, Fungi
Ref: Jawetz Microbiology, 27th edition, page 691
Pneumocystis jiroveci is a causative agent of pneumonia in immunocompromised patients. For years, P. jiroveci was
thought to be a protozoan but molecular biologic studies have proved that it is a fungus with a close relationship to
ascomycetes. Pneumocystis species are present in the lings of many animals (rats, mice, dogs, cats, ferrets, rabbits) but
rarely cause disease unless the host is immunosuppressed.

174. A patient receives a renal transplant from an identical twin sister, what type of graft is it?
a. Allograft
b. Autograft
c. Isograft
d. Xenograft

Ans: C, Isograft
Ref: Ananthanarayan Microbiology, 10th edition, page 191
 Transplants may be classified based on the organs or tissue transplanted, the anatomical site of origin of
the transplant and its placement. Based on the genetic relationships, transplants can be classified as:
o autograft (taken from an individual and grafted on himself/herself)
o isograft (between individuals of the same genetic constitution, like identical twins)
o allograft (between two genetically non-identical members of the same species)
o xenograft (between members of different species)

175. What is Burkitt's lymphoma associated with?


a. EB virus
b. Hep B virus
c. HPV virus
d. HTL virus

Ans: A, EB virus
Ref: Ananthanarayan Microbiology, 10th edition, page 570
 Epstein-Barr virus is regularly found in cultured lymphocytes from Burkitt's lymphoma patients.
 In the body, the tumour cells contain no virus, but cell lines established from them contain 5-20 per cent
of cells that produce the virus.

Community Medicine
176. Which of the following is not included in IMNCI?
a. Diarrhea
b. Measles

82
ONLINE PG NEPAL QAE Magh 1

c. Malnutrition
d. Chicken pox

Ans: d
IMNCI is an integration of CB-IMCI and CB-NCP Programs as per the decision of MoH on 2071/6/28 (October 14, 2014).
This integrated package of child-survival intervention addresses the major problems of sick newborn such as birth
asphyxia, bacterial infection, jaundice, hypothermia, low birthweight, counseling of breastfeeding.
It also maintains its aim to address major childhood illnesses like Pneumonia, Diarrhea, Malaria, Measles and Malnutrition
among under 5 year’s children in a holistic way.

177. Vector Borne Disease Research & Training includes:


a. Malaria
b. Chikungunya
c. Scrub typhus
d. All of the above

Ans: d
Vector Borne Disease Research & Training
The objective of Vector Borne Disease Research and Training Center is primarily to fill the knowledge gap and generate
scientific evidences in the field of Vector Borne Diseases. Therefore, VBDRTC is responsible for researches and trainings
that relate with VBDs such as Malaria, Kala-azar, Dengue, Chikungunya, Lymphatic filariasis, Scrub typhus and Japanese
encephalitis.

178. Nepal was certified polio free in:


a. 2006
b. 2010
c. 2014
d. 2016

Ans: c
 The last case of polio in Nepal was reported in August 2010. Along with the other countries in the South East Asia
Region, Nepal was certified polio free in 2014.
 Since then, Nepal has maintained this status.
 For sensitive surveillance of polio, there are two main cardinal indicators: 1) non-polio AFP rate which should be at
least 2 per 100,000 under 15 years population, and 2) adequate stool collection rate which should be 80% or
more.

179. The primary goal of Human Genome Project has been:


a. Introduction of a gene sequence into a cell to modify its behavior
b. Development of new diagnostic techniques such as restriction enzymes
c. Identify genes and sequence of base pairs in DNA of human genome
d. Confirmation of Hardy Weinberg Law

Ans: c
Ref: Park's Textbook of Preventive and Social Medicine; 24th Edition; Page No: 863

83
ONLINE PG NEPAL QAE Magh 1

The human genome project


The human genome project is an attempt to systematize the research on mapping and isolating human genes that is
already in progress in many countries, in order to create a single linear map of the human genome, with each coding gene
defined and sequenced.
Agencies with a role in coordinating human genome data include UNESCO, the Genome Data Base, HUGO, the National
Institute of Health/Department of Energy (USA), the Medical Research Council (UK), Genethon (France) and the European
Union.

180. Income generated within a country is known as:


a. Gross Domestic Product (GDP)
b. Net National Product (NNP)
c. Net Domestic Product (NDP)
d. Purchasing Power Parity (PPP)

Ans: a
Ref: Park's Textbook of Preventive and Social Medicine; 24th Edition; Page No: 737
 Per capita income: An index of the standard of living of the people is "per capita income". Per capita income in
India is among the lowest in the world.
 Gross domestic product (GDP): GDP is gross income generated within a country, i.e., it excludes net income
received from abroad.
 Net National Product (NNP): It is the CNP minus the capital we consume (e.g., equipment, machinery, etc.,) in the
production process. In other words, NNP is the market value of all final goods and services after providing for
depreciation.

181. Salter's Scale is a useful method employed in the field to measure:


a. Mid arm Circumference
b. Length at birth
c. Skin fold thickness
d. Birth weight

Ans: d
Ref: Park's Textbook of Preventive and Social Medicine; 24th Edition; Page No: 569
 The birth-weight should be taken preferably within the first hour of life before significant post-natal weight loss
has occurred. The naked baby should be placed on a clean towel on the scale pan. In home delivery, weight is
taken by placing the baby in a sling bag using a salter weighing scale.
 The child is weighed to the nearest 100 g according to the standard method for weight measurement.

Research methodology
182. Formulative research study is:
a. To gain familiarity with a phenomenon or to achieve new insights into it
b. To portray accurately the characteristics of a particular individual, situation or a group
c. To determine the frequency with which something occurs or with which it is associated with something else
d. To test a hypothesis of a causal relationship between variables

Ans: a

84
ONLINE PG NEPAL QAE Magh 1

Ref: Kothari Research Methodology, 2nd Revised Edition, Page No: 2


The purpose of research is to discover answers to questions through the application of scientific procedures. The main aim
of research is to find out the truth which is hidden and which has not been discovered as yet. Though each research study
has its own specific purpose, we may think of research objectives as falling into a number of following broad groupings:
 To gain familiarity with a phenomenon or to achieve new insights into it (studies with this object in view are
termed as exploratory or formulative research studies);
 To portray accurately the characteristics of a particular individual, situation or a group (studies with this object in
view are known as descriptive research studies);
 To determine the frequency with which something occurs or with which it is associated with something else
(studies with this object in view are known as diagnostic research studies);
 To test a hypothesis of a causal relationship between variables (such studies are known as hypothesis-testing
research studies).

183. Quality of a good research is not:


a. Good research is systematic
b. Good research is empirical
c. Good research is replicable
d. Good research is easy to conduct

Ans: d
Ref: Kothari Research Methodology, 2nd Revised Edition, Page No: 21
Qualities of a good research:
 Good research is systematic: It means that research is structured with specified steps to be taken in a specified
sequence in accordance with the well defined set of rules. Systematic characteristic of the research does not rule
out creative thinking but it certainly does reject the use of guessing and intuition in arriving at conclusions.
 Good research is logical: This implies that research is guided by the rules of logical reasoning and the logical
process of induction and deduction are of great value in carrying out research. Induction is the process of
reasoning from a part to the whole whereas deduction is the process of reasoning from some premise to a
conclusion which follows from that very premise. In fact, logical reasoning makes research more meaningful in the
context of decision making.
 Good research is empirical: It implies that research is related basically to one or more aspects of a real situation
and deals with concrete data that provides a basis for external validity to research results.
 Good research is replicable: This characteristic allows research results to be verified by replicating the study and
thereby building a sound basis for decisions.

184. A physician, after examining a group of patients of a certain disease, classifies the condition of each one as
'Normal', 'Mild', 'Moderate' or 'Severe'. Which one of the following is the scale of measurement that is being adopted
for classification of the disease condition?
a. Normal
b. Interval
c. Ratio
d. Ordinal

Ans: d
Ref: Kothari Research Methodology, 2nd Revised Edition, Page No: 71

85
ONLINE PG NEPAL QAE Magh 1

Ordinal scale:
 The lowest level of the ordered scale that is commonly used is the ordinal scale.
 The ordinal scale places events in order, but there is no attempt to make the intervals of the scale equal in terms
of some rule. Rank orders represent ordinal scales and are frequently used in research relating to qualitative
phenomena.
 A student’s rank in his graduation class involves the use of an ordinal scale. One has to be very careful in making
statement about scores based on ordinal scales.
 For instance, if Ram’s position in his class is 10 and Mohan’s position is 40, it cannot be said that Ram’s position is
four times as good as that of Mohan. The statement would make no sense at all.
 Ordinal scales only permit the ranking of items from highest to lowest. Ordinal measures have no absolute values,
and the real differences between adjacent ranks may not be equal. All that can be said is that one person is higher
or lower on the scale than another, but more precise comparisons cannot be made.
 Thus, the use of an ordinal scale implies a statement of ‘greater than’ or ‘less than’ (an equality statement is also
acceptable) without our being able to state how much greater or less. The real difference between ranks 1 and 2
may be more or less than the difference between ranks 5 and 6.
 Since the numbers of this scale have only a rank meaning, the appropriate measure of central tendency is the
median. A percentile or quartile measure is used for measuring dispersion. Correlations are restricted to various
rank order methods. Measures of statistical significance are restricted to the non-parametric methods.

185. Stable or reproducible criterion is:


a. Relevance
b. Freedom from bias
c. Reliability
d. Availability

Ans: c
Ref: Kothari Research Methodology, 2nd Revised Edition, Page No: 74
Criterion-related validity relates to our ability to predict some outcome or estimate the existence of some current
condition. This form of validity reflects the success of measures used for some empirical estimating purpose.
The concerned criterion must possess the following qualities:
 Relevance: A criterion is relevant if it is defined in terms we judge to be the proper measure.
 Freedom from bias: Freedom from bias is attained when the criterion gives each subject an equal opportunity to
score well.
 Reliability: A reliable criterion is stable or reproducible.
 Availability: The information specified by the criterion must be available.

Forensic Medicine
186. Blanket consent is consent taken:
a. When the patient comes to doctor for treatment
b. Orally when given during any procedure
c. Implied by gesture
d. At the time of admission to do any surgery

Ans: d
Ref: Gautam Biswas Forensic Medicine, 3rdEdition, Page No: 25

86
ONLINE PG NEPAL QAE Magh 1

 Substituted consent: If a person in need of treatment is incapable of giving informed consent, consent (proxy
consent) must be obtained from next of kin. The order of succession is generally spouse, adult child, parent and
sibling.
 Blanket (open) consent: The consent taken at the time of admission, and practiced in most hospitals that cover
almost everything a doctor might do to a patient without mentioning anything specific. It is of questionable legal
validity.
 Presumed consent assumes that an individual agrees in principle to the said procedure; if not, he/she must
withdraw his/her consent, i.e. ‘opt out’.

187. Vicarious responsibility pertains to:


a. Patient’s, contribution towards negligence
b. Hospitals contribution towards patient’s damage
c. Responsibility for actions of a colleague
d. Responsibility of senior for actions of junior

Ans: d
Ref: Gautam Biswas Forensic Medicine, 3rdEdition, Page No: 21
Vicarious Liability/Respondent Superior
An employer is responsible not only for his own negligent act, but also for the negligent act of his employees by the
principle of ‘respondent superior’ (Latin, ‘let the master answer’), if three conditions are satisfied:
 There must be an employer-employee relationship
 The employee’s conduct must occur within the scope of his employment
 Incident must occur while on the job.
It also called the ‘Master-Servant Rule’.

188. Agonal period is the time between:


a. Entry into the ventilator to death
b. Lethal occurrence and death
c. Birth and death
d. Trauma and recovery

Ans: b
Ref: Gautam Biswas Forensic Medicine, 3rdEdition, Page No: 128
Agonal period is the time between a lethal occurrence and death.

189. Tattooing in entry wound of a firearm injury is due to:


a. Burns
b. Smoke
c. Gunpowder
d. Wads

Ans: c
Ref: Gautam Biswas Forensic Medicine, 3rdEdition, Page No: 216
Tattooing: It consists of unburnt or partially burnt powder particles that are embedded in and under the skin through the
force of their impact (when the weapon is near enough for the powder grains to strike).

87
ONLINE PG NEPAL QAE Magh 1

 Tattooing is an antemortem phenomenon and indicates that the individual was alive; and it cannot be wiped away
with a wet cotton.
 It consists of numerous reddish-brown punctate abrasions surrounding the wound of entrance.
 The greater the range, the larger and less dense the powder tattooing.
 Marks usually heal completely if the individual survives (involves the superficial layer of the epidermis).

190. An infant is brought to casualty with reports of violent shaking by parents. Most characteristic injury is:
a. Long bone fracture
b. Ruptured spleen
c. Subdural hematoma
d. Skull bone fracture

Ans: c
Ref: Gautam Biswas Forensic Medicine, 3rdEdition, Page No: 333
Shaken baby syndrome:
 Infants are susceptible to subdural/ subarachnoid hematoma and retinal hemorrhages due to vigorous shaking of
the baby as a method of punishment.
 Predisposing factors: Infant’s relatively large head, weak neck muscles and delicate subarachnoid bridging vessels.
 Signs and symptoms: Seizures, irritability, meningismus and focal or general neurologic deficit.
 Diagnosis: Confirmation by CT/MRI scan, bloody spinal or subdural fluid and normal skull X-rays.

Mandatory CPD Topics


191. How many of the districts of Nepal are declared Full immunized?
a. 18
b. 34
c. 58
d. 70

Ans: c
As of end of FY 2076/77, 58 out of 77 districts have been declared ‘fully immunized’. Gandaki Province has had declared
their province as fully immunized province.

192. Which of the following is not true regarding COVID 19?


a. There is no evidence from the current outbreak that eating garlic has protected people from the new coronavirus.
b. Antibiotics should not be used for prevention or treatment of COVID19
c. UV radiation of hands can be an effective way to sanitize hands against coronavirus
d. Hand dryers are not effective in killing the COVID-19 virus

Ans: c
 Garlic is a healthy food that may have some antimicrobial properties. However, there is no evidence from the
current outbreak that eating garlic has protected people from the new coronavirus.
 Antibiotics work only against bacteria, not viruses. COVID-19 is caused by a virus, and therefore antibiotics should
not be used for prevention or treatment.
 UV radiation can cause skin irritation and damage your eyes. Cleaning your hands with alcohol-based hand rub or
washing your hands with soap and water are the most effective ways to remove the virus.

88
ONLINE PG NEPAL QAE Magh 1

 Hand dryers are not effective in killing the COVID-19 virus. To protect yourself, frequently clean your hands with
an alcohol-based hand rub or wash them with soap and water. Once your hands are cleaned, you should dry them
thoroughly by using paper towels or a warm air dryer.

193. Separating and restricting the movement of people who were exposed to a contagious disease to see if they
become sick is:
a. Isolation
b. Protection
c. Quarantine
d. Separation

Ans: c
 Isolation separates sick people with a contagious disease from people who are not sick.
 Quarantine separates and restricts the movement of people who were exposed to a contagious disease to see if
they become sick.

194. Filters used in modern surgical masks and respirators acts by:
a. Inertial impaction
b. Interception
c. Electrostatic attraction
d. All of the above

Ans: d
The filters used in modern surgical masks and respirators are considered “fibrous” in nature—constructed from flat,
nonwoven mats of fine fibers.
How do filters collect particles? These capture, or filtration, mechanisms are described as follows:
 Inertial impaction: With this mechanism, particles having too much inertia due to size or mass cannot follow
the airstream as it is diverted around a filter fiber. This mechanism is responsible for collecting larger particles.
 Interception: As particles pass close to a filter fiber, they may be intercepted by the fiber. Again, this
mechanism is responsible for collecting larger particles.
 Diffusion: Small particles are constantly bombarded by air molecules, which causes them to deviate from the
airstream and come into contact with a filter fiber. This mechanism is responsible for collecting smaller
particles.
 Electrostatic attraction: Oppositely charged particles are attracted to a charged fiber. This collection
mechanism does not favor a certain particle size.

195. Which of the following is irrational use of drug?


a. Giving drugs for Syndromic STI approach
b. Giving antibiotics for common cold
c. Giving antiallergic drugs for allergic rhinitis
d. Giving antibiotics blanket for Trachoma

Ans: b

89
ONLINE PG NEPAL QAE Magh 1

196. In addition to having the right to refuse a diagnostic or therapeutic intervention, patients also have the right to
refuse to receive information. This is based upon which ethical principle?
a. Autonomy
b. Beneficence
c. Nonmaleficence
d. Justice

Ans: a
Autonomy
 Respect patients as individuals (e.g., respecting their privacy by maintaining confidentiality and being truthful
about their medical care).
 Provide the information and opportunity for patients to make their own decisions regarding their care (e.g.,
informed consent).
 Honor and respect patients' decisions regarding their choice to accept or decline care.
 In addition to having the right to refuse a diagnostic or therapeutic intervention, patients also have the right to
refuse to receive information
Beneficence
 Act in the best interest of the patient and advocate for the patient.
 May conflict with autonomy
Nonmaleficence
 Avoid causing injury or suffering to patients
 May conflict with beneficence
Justice
 Treat patients fairly and equitably.
 Equity is not the same as equality.

197. “Systematic interactive forecasting method” for obtaining consensus forecasts from a panel of independent
geographically dispersed experts is:
a. Symposium
b. Panel discussion
c. Delphi method
d. Focus Group Discussion

Ans: c
Delphi method:
“Systematic interactive forecasting method” for obtaining consensus forecasts from a panel of independent geographically
dispersed experts
Method:
 Carefully selected experts answer questionnaires in two or more rounds
 After each round, a facilitator provides an anonymous summary of the experts’ forecasts from the previous round
as well as the reasons they provided for their judgments
 Thus, participants are encouraged to revise their earlier answers in light of the replies of other members of the
group
 Range of the answers decrease and the group will converge towards the ‘correct’ consensual answer

90
ONLINE PG NEPAL QAE Magh 1

 Finally, the process is stopped after a pre-defined stop criterion (e.g. number of rounds, achievement of
consensus, stability of results) and the mean or median scores of the final rounds determine the results

198. Immediately after birth, an infant in need of resuscitation should be:


a. Placed under a radiant heater and dried
b. Airway should be cleared by suctioning
c. Gentle tactile stimulation provided (slapping the foot, rubbing the back)
d. All of the above

Ans: d
Immediately after birth, an infant in need of resuscitation should be placed under a radiant heater and dried (to avoid
passive hypothermia), positioned with the head down and slightly extended; the airway should be cleared by suctioning,
and gentle tactile stimulation provided (slapping the foot, rubbing the back). Simultaneously, the infant’s color, heart rate,
and respiratory effort should be assessed.

199. Regarding CPR, true is:


a. Anesthetic agents are not usually required
b. Sedation should be done
c. Both Anesthesia and sedation required
d. Local anesthesia can be given

Ans: a
Because a person in cardiac arrest is almost invariably unconscious, anesthetic agents are not typically required for
cardiopulmonary resuscitation (CPR).

200. You are a resident doctor required to attend mandatory teaching on Tuesday afternoon. This is also the time that
your consultant holds his only ward round of the week. Your registrar is unimpressed that you want to ‘slip off’ when
you are needed to update the consultant on each patient’s progress. He suggests that you sign the attendance register
then return to the ward round. What will you do?
a. Speak with your consultant and explain that your commitments are conflicting.
b. Sign the attendance register so that your progression through FY1 is not obstructed, and then attend the ward
round.
c. Agree that your presence on the ward round is necessary and that you will miss teaching.
d. Send a text message asking an FY1 colleague to sign you in to teaching.

Ans: a
It is your responsibility to satisfy all mandatory requirements of the Residency program. However, resident doctors are a
key part of the clinical team and your absence could impact negatively on patient care. Therefore you should take steps to
minimize the impact of your absence. You should attend mandatory commitments unless this compromises patient safety.
However, to resolve a potential conflict in future, you should seek advice from your consultant.

NEXT MD/MS BASED MODEL TEST


07:00 PM - 09:00 PM, Magh 3, 2078 (Monday evening): USMLE Step 1 First Aid Based (100 marks) [Online Only]
07:00 PM - 10:00 PM, Magh 5, 2078 (Wednesday evening): CEE (200 marks) MD/MS Based [Online Only]

91
ONLINE PG NEPAL QAE Magh 1

07:00 PM - 10:00 PM, Magh 6, 2078 (Thursday evening): Past Question Based (200 marks) [Online Only]

Real CEE MD/MS Entrance: 11:00 AM - 02:00 PM, Magh 8, 2078 (Saturday noon)

Visit our Website for Online Test: www.onlinepgnepal.com

BEST WISHES

92

You might also like